Anda di halaman 1dari 224

SBMA2203

DIFFERENTIAL
CALCULUS
Dr Hafsah Abd Hamid
Muriati Mukhtar
Siti Aisyah Hanawi
Dr Maslina bt Darus

Project Directors:

Prof Dr Mansor Fadzil


Prof Dr Latifah Abdol Latif
Open University Malaysia

Module Writers:

Dr Hafsah Abd Hamid


Muriati Mukhtar
Siti Aisyah Hanawi
Fakulti Teknologi dan Sains Maklumat
Universiti Kebangsaan Malaysia
Dr Maslina bt Darus
Fakulti Sains dan Teknologi
Universiti Kebangsaan Malaysia

Moderators:

Prof Abu Osman Md Tap


Universiti Kebangsaan Malaysia
Puan Raziana Che Aziz
Assoc Prof Dr Maselan @ Mazelan Ali
Open University Malaysia

Translator:

Assoc Prof Dr Mohd Idris Jayes


Open University Malaysia

Developed by:

Centre for Instructional Design and Technology


Open University Malaysia

Printed by:

Meteor Doc. Sdn. Bhd.


Lot 47-48, Jalan SR 1/9, Seksyen 9,
Jalan Serdang Raya, Taman Serdang Raya,
43300 Seri Kembangan, Selangor Darul Ehsan

First Printing, January 2004


Tenth Printing, August 2010
Eleventh Printing, October 2010
Twelfth Printing, October 2011
Copyright Open University Malaysia (OUM), October 2011, SBMA2203
All rights reserved. No part of this work may be reproduced in any form or by any means
without the written permission of the President, Open University Malaysia (OUM).
Version October 2011

Table of Contents

Course Guide

vii-xii

Topic 1

Function and Graph


1.1 Function and Graph of a Function
1.2 Symmetry Property and Shifting
Summary

1
2
14
18

Topic 2

Limit
2.1 Limit Through the Graph of a Function
2.2 Limit Through an Approximate Value
2.3 Limit Through Definition
2.4 Rules of Calculating Limit
2.5 Infinite Limit
Summary

20
21
25
29
38
46
52

Topic 3

Continuity
3.1 The Meaning and Properties of a Continuous Function
3.2 Mid Value Theorem
Summary

53
53
62
65

Topic 4

Tangent Line and Derivative


4.1 Instantaneous Speed and Slope of Tangent Line
4.1.1 Average Speed
4.1.2 Instantaneous Speed
4.1.3 Instantaneous Speed Using Graph of a Function
4.2 Derivatives
4.3 Derivatives of Some Primary Functions
Summary

66
67
67
68
70
73
84
89

Topic 5

Differentiation Rule
5.1 General Formula of Differentiation
5.2 The Chain Rule
5.3 Implicit Differentiation
Summary

90
90
98
105
110

Topic 6

Theorems on Differentiable Function


6.1 Theorems of the Derivatives of an Inverse Function
6.2 Mean Value Theorem and Rolles Theorem
Summary

111
111
115
122

iv

TABLE OF CONTENTS

Topic 7

Various Rates of Change


7.1 Rate of Change
Summary

123
123
130

Topic 8

Application in Economics
8.1 Marginal Profit and Cost Functions
Summary

131
131
136

Topic 9

Approximation
9.1 Differential
9.2 Linear Approximation
9.3 Taylor and Quadratic Approximation
9.4 Newton Method
Summary

137
137
142
145
150
154

Topic 10 Extremum Problem and the Tracing of a Graph


10.1 Extremum and Local Extremum
10.2 Graph Tracing
10.3 Applied Extremum Problem
Summary

155
155
166
178
181

Topic 11 LHospitals Rule


11.1 Determination of Limit
11.2 The Lhospitals Rule Method
Summary

182
182
183
186

Answers

187

COURSE GUIDE

COURSE GUIDE DESCRIPTION


You must read this Course Guide carefully from the beginning to the end. It tells
you briefly what the course is about and how you can work your way through the
course material. It also suggests the amount of time you are likely to spend in
order to complete the course successfully. Please keep on referring to Course
Guide as you go through the course material as it will help you to clarify
important study components or points that you might miss or overlook.

INTRODUCTION
SBMA2203 Differential Calculus is one of the courses offered by the Faculty of
Science and Technology, Open University Malaysia (OUM). This is a 3 credit
hour course that spans over 15 weeks and is usually offered twice per year
(January Semester and September Semester).

COURSE AUDIENCE
This course is offered to all students taking the bachelor programme where
mathematics is a compulsory course. This module aims to impart the knowledge
of mathematics.
As an open and distance learner, you should be acquainted with learning
independently and being able to optimise the learning modes and environment
available to you. Before you begin this course, please confirm the course material,
the course requirements and how the course is conducted.

STUDY SCHEDULE
It is a standard OUM practice that learners accumulate 40 study hours for every
credit hour. As such, for a three-credit hour course, you are expected to spend 120
study hours. Table 1 gives an estimation of how the 120 study hours could be
accumulated.

viii X

COURSE GUIDE

Table 1: Estimation of Time Accumulation of Study Hours


STUDY ACTIVITIES

STUDY
HOURS

Briefly go through the course content and participate in initial


discussions

Study the module

60

Attend tutorial sessions

10

Online Participation

12

Revision

15

Assignment(s), Test(s) and Examination(s)

20

TOTAL STUDY HOURS ACCUMULATED

120

COURSE OBJECTIVES
By the end of this course, you should be able to:
1.

Understand the concept of limit, continuity and derivative of a function;

2.

Sketch the graphs of some functions;

3.

Determine the existence of limit and continuity of a function;

4.

Approximate the value of a limit of a function;

5.

Prove existence of a limit of a given function;

6.

Determine the continuity of a function at a given point or in given interval;

7.

Calculate the limit of a function;

8.

Apply the rules of calculating limit;

9.

Apply the Mid Value Theorem to solve a mathematical problem;

10.

Determine the derivative of a function from the first principle;

11.

Use the rules of differentiation; and

12.

Apply the differentiation concept to solve some practical problems.

COURSE GUIDE

ix

COURSE SYNOPSIS
This course is divided into 11 topics. The synopsis for each topic can be listed as
follows:
Topic 1 starts with graphs of various functions which form the basis in the study
of calculus. For example, the graphs of power function and exponential function
as well as its inverse, that is the logarithmic function. The graphs of trigonometric
functions are also considered. Next, the symmetrical properties of graphs and the
effects of vertical transformation are explained.
Topic 2 discussed the concept of limit is elaborately in since it is an important
element in calculus. We shall first use the geometrical approach that is we state
the value of limit through a graph. In the case where a graph is not provided, we
shall normally predict the value of a limit using an approximate value. And
illustrate the use of the formal approach in the study of limit that is through the
definition of limit. We shall illustrate several proofs that involve limit.Next; we
state several rules for the calculation of limit followed by idea of infinite limit as
to complete the study of the limit.
Topic 3 introduced the concept of continuity of a function. It starts with the
discussion of continuity at a point where it involves both the limit and value of a
given function at the point under consideration. Next, we shall give several simple
rules that can be adopted to distinguish the continuity of a composite function.
The topic ends with a theorem which states the properties of a continuous
function.
Topic 4 observe that some of the continuous functions are differentiable or said to
have derivatives. The concept of derivative involves an average of change in an
interval and the limit is calculated as the width of the interval to zero. Topic 4
ends with a process of determining the derivatives of several primary functions.
Topic 5 will be given the general formula of differentiation and the Chain Rule
that simplifies the determination of the derivatives of the combination and
composition of two functions. The determination of the implicit form of the
derivative of a function is also demonstrated.
Topic 6 an easy way of determining the derivative of an inverse function without
first obtaining the inverse function itself is explained. The theorem that explains
how to achieve the average change in an interval is also discussed.

COURSE GUIDE

Topic 7 will consider a simple problem with a single variable. For example, the
problem of the motion of several objects with an uniform speed and the problem
of a population growth.
Topic 8 will learn about the application in economics with the quantity of goods
as the variable.
Topic 9 will learn about the application of derivative in approximation, such as
the problem of obtaining the approximate value of a certain function or the
problem of approximating the roots of some functions.
Topic 10 discuss the problem of maximum and minimum and the method of
determining the location of the extremum point. This is further extended to the
application of derivative in tracing the graph of a function.
Topic 11 discuss the application of derivative in mathematics itself and in
particular the LHospitals Rule.

TEXT ARRANGEMENT GUIDE


Before you go through this module, it is important that you note the text
arrangement. Understanding the text arrangement should help you to organise
your study of this course to be more objective and more effective. Generally, the
text arrangement for each topic is as follows:
Learning Outcomes: This section refers to what you should achieve after you
have completely gone through a topic. As you go through each topic, you should
frequently refer to these learning outcomes. By doing this, you can continuously
gauge your progress of digesting the topic.
Self-Check: This component of the module is inserted at strategic locations
throughout the module. It is inserted after you have gone through one sub-section
or sometimes a few sub-sections. It usually comes in the form of a question that
may require you to stop your reading and start thinking. When you come across
this component, try to reflect on what you have already gone through. When you
attempt to answer the question prompted, you should be able to gauge whether
you have understood what you have read (clearly, vaguely or worse you might
find out that you had not comprehended or retained the sub-section(s) that you
had just gone through). Most of the time, the answers to the questions can be
found directly from the module itself.
Activity: Like Self-Check, activities are also placed at various locations or
junctures throughout the module. Compared to Self-Check, Activity can appear in
various forms such as questions, short case studies or it may even ask you to

COURSE GUIDE

xi

conduct an observation or research. Activity may also ask your opinion and
evaluation on a given scenario. When you come across an Activity, you should try
to widen what you have gathered from the module and introduce it to real
situations. You should engage yourself in higher order thinking where you might
be required to analyse, synthesise and evaluate instead of just having to recall and
define.
Summary: You can find this component at the end of each topic. This component
helps you to recap the whole topic. By going through the summary, you should be
able to gauge your knowledge retention level. Should you find points inside the
summary that you do not fully understand, it would be a good idea for you to
revisit the details from the module.
Key Terms: This component can be found at the end of each topic. You should
go through this component to remind yourself of important terms or jargons used
throughout the module. Should you find terms here that you are not able to
explain, you should look for the terms from the module.
References: References is where a list of relevant and useful textbooks, journals,
articles, electronic contents or sources can be found. This list can appear in a few
locations such as in the Course Guide (at References section), at the end of every
topic or at the back of the module. You are encouraged to read and refer to the
suggested sources to elicit the additional information needed as well as to enhance
your overall understanding of the course.

PRIOR KNOWLEDGE
The students should have followed and have a good knowledge of the basic
mathematics course or a good grade in the SPM Additional Mathematics or its
equivalents.

ASSESSMENT METHOD
Please refer to myVLE for the latest assessment method.

xii X

COURSE GUIDE

REFERENCES
Abu Bakar Haji Musa (1998). Kalkulus Awalan. Penerbit Universiti Putra
Malaysia.
Anton, H. (1999). Calculus - A New Horizon (5th ed.). John Wiley & Sons, Inc.
Anton, H., Bivens, L., & Davis, S. (2002). Calculus (7th ed.), John Wiley & Sons,
Inc.
Apostol, T. M. (1967). Calculus (2nd ed.). New York: John Wiley & Sons, Inc..
Larry, J. G., David, C. L., & David, I. S. (2001). Calculus and its Applications.
Prentice Hall.
Larson, R. E., Hostetler, R. P., & Edwards, B. H. (1998). Calculus with Analytic
Geometry (6th ed.). Houghton Mifflin.
Stewart, J. (2003). Single Variable Calculus Early Transcendentals (5th ed.).
Thmson.

Topic X Function and

Graph

LEARNING OUTCOMES
By the end of this topic, you should be able to:

1.

Distinguish the differences between the graph of a function and the


graph of a non function;

2.

Sketch the 4 graphs of the primary functions, namely, the constant


function, the linear function, the modulus function and the piecewise
function; and

3.

Recognise the symmetry properties and the shifting of the graph of a


function.

INTRODUCTION

A function is a relation between the domain and the range of a given number. In a
function, if a certain number is given, then the value that relates the given number
will also be known. Firstly, you have to know the graphs of a linear equation,
circle and parabola. From these equations, you will then be able to relate the
relationships between the dependant and the independant variables of a function.

X TOPIC 1

1.1

FUNCTION AND GRAPH

FUNCTION AND GRAPH OF A FUNCTION

Function
You need to be familiar and be able to recognise the various types of equations
and their graphs before you proceed further. Let us review the following
equations:
(a)

2
y = x + 1 is the equation of a straight line.
3

(b)

x2 = y is the equation of a parabola.

If x is substituted by a certain value in (a), we will notice that y will have a certain
value.
Similarly, if we replace some values of +x and x in (b), we will obtain a value of
y. Please try!
We are able to recognise 2 types of relations that associate x with y, they are:
(i)

2
one to one such as in equation y = x + 1 ; and
3

(ii)

many to one such as in equation x2 = y.

The relations one to one and many to one are called function.
However, there is an equation that associates x with y by the relation one to many.
Example 1.1
x2 + y2 = 25 is the equation of a circle.
If x = 4 is substituted in the above equation, we obtain y = 3 or y = 3.
Example 1.2
y2 = x is the equation of a parabola.
If we substitute x = 4, we obtain y = 2 or y = 2.

TOPIC 1

FUNCTION AND GRAPH W

Therefore, the value of y that is associated with a particular value of x is not


unique. Thus Example 1.1 and Example 1.2 above do not meet the criteria of a
function.
It has become conventional for mathematician to write a function in a special
2
notation. For example, the equation y = x + 1 can be written in the form y = f (x)
3
where the function f is given by

2
f ( x) = x + 1 .
3
Such notation was introduced by Leonhard Euler in 1734 and the word function
was first used by Gottfried Wilhelm Leibniz in 1694.
EXERCISE 1.1
1.

Determine whether the following diagrams represent graph of a


function or graph of an equation.

(a)

(b)

(c)

(d)

X TOPIC 1

FUNCTION AND GRAPH

Graph of a Function
As in the graph of an equation, the graph of a function f is a set of points
{(x, f (x)) where x is in the domain of f }.

Figure 1.1: Graph of y2 = x

Figure 1.1 shows the vertical line x = 4, cuts the curve that represents the graph of
the equation y2 = x at more than one points. This situation shows that the graph
drawn does not represent the graph of a function.
Now we list several graphs which you have to know such as the graph of a
constant function, graph of a linear function, graph of a modulus function and the
graph of a piecewise function as in Figure 1.2a, Figure 1.2b, Figure 1.2c and
Figure 1.2d.

Figure 1.2a: Graph of a constant function

TOPIC 1

FUNCTION AND GRAPH W

Figure 1.2b: Graph of a linear function

Figure 1.2c: Graph of a modulus function

Figure 1.2d: Graph of a piecewise function

X TOPIC 1

FUNCTION AND GRAPH

Note that Figure 1.2d represents graph of a function


1 ,
f ( x) =
1 ,

x>0
x<0

which is usually written as

f ( x) =

x
x

, x0

Besides the above functions, there are other examples of graphs of functions such
as the graph of a quadratic function and graph of a cubic function as shown in
Figure 1.3a and Figure 1.3b.

Figure 1.3a: Graph of a quadratic function

TOPIC 1

FUNCTION AND GRAPH W

Figure 1.3b: Graph of a cubic function

However, generally the graph of a quadratic function is of the form as shown in


Figure 1.4.

Figure 1.4: General graph of a quadratic function

X TOPIC 1

FUNCTION AND GRAPH

The graph of a cubic is generally of the form as given in Figure 1.5.

Figure 1.5: General graph of a cubic function

Next, the graph of a polynomial function of degree n can have at most n1


turning points. For example,
f (x) = x 3 can have only 1 turning point
while
f (x) = x 3 x has 2 turning points.
However, the graph of the polynomial
f (x) = an xn + an1 xn1 + ... + a2 x2 + a1x + a0
finally continues to ascend or descend depending on the order of the polynomial
(n is either even or odd) and the sign of the leading coefficient an. Figure 1.6
illustrates several graphs of the polynomial function.

TOPIC 1

FUNCTION AND GRAPH W

Figure 1.6: Graph of a polynomial function


1

How about the graph of the nth root of a function, such as f ( x) = x n = n x ? If n is


1
2

even, say f ( x) = x the graph is of the form as in Figure 1.7a.

Figure 1.7a: Graph of the function f ( x) = x 2

10

X TOPIC 1

FUNCTION AND GRAPH

When n is odd, say g ( x) = x 3 , the graph is of the form as given in Figure 1.7b.

Figure 1.7b: Graph of the function g ( x) = x 3

Notice the distinction between the domain and the range of the function
1

f ( x) = x 2 and the domain and the range of the function g ( x) = x 3 .


Next, we consider the graph of the function f ( x) = x n =
example, f ( x) =

1
. If n is odd, for
xn

1
, the graph is of the form given in Figure 1.8a.
x

Figure 1.8a: Graph of f ( x) =

1
x

TOPIC 1

FUNCTION AND GRAPH W

11

If n is even, for example, the graph is as shown in Figure 1.8b.

Figure 1.8b: Graph of g ( x) =

1
x2

Here we have to consider the distinction between the range of the function
1
1
f ( x) = and the range of the function g ( x) = 2 .
x
x
Next, the graph of exponential function f ( x) = e x and its inverse function
g(x) = ln x, is given in Figure 1.9a and Figure 1.9b, respectively.

Figure 1.9a: Graph of (x) = e x

12

X TOPIC 1

FUNCTION AND GRAPH

Figure 1.9b: Graph of g ( x) = ln x

Finally, note the graphs of the various trigonometric functions, especially the sine,
cosine and tangent functions as shown in Figure 1.10.

Figure 1.10: Graph of the trigonometric functions

TOPIC 1

FUNCTION AND GRAPH W

13

Remember that we may not be able to draw the graphs of all functions. For
example, we cannot draw the graph of the Dirichlet function
1, x is rational
f (x) =
0, x is non rational
Instead, we may sometime be able to draw the graph even though the function
may be difficult to formulate. For example, suppose we are filling an empty
beaker with water at a constant rate. The height of the water in the beaker is a
function of the variable time, x.

ACTIVITY 1.1
Observe the animation in the website (http://www.ies.co.jp/
math/java/geo/lin_line/lin_line.html). Compare the illustration from
the animation in the website with the graph of the height of water
illustrated in the following figure.

The graph of the water height can be sketched, but the function is quite difficult to
be obtained.

Figure 1.11: Graph of the height of water

By referring to Figure 1.11, we observe that initially, the rate at which the height
of water rises is slowing down (since the area is getting large). Next, the rate is
increasingly fast (since the surface area is getting small). Finally, the rate slows
down but is not as slow as in the initial stage.

14

X TOPIC 1

FUNCTION AND GRAPH

SELF-CHECK 1.1
By referring to Figure 1.11, can you explain the reason in the change
of the form of the resulting graph.

Note also that if we consider the width of the graph with respect to the y-axis, we
shall obtain the range of the function. The width of the graph with respect to the
x-axis gives the domain of the function.

EXERCISE 1.2
Based on the graphs of the quadratic and cubic functions, sketch the
graphs of f ( x) = x x and g ( x) = x 2 x .

Hint :Recapitilate

1.2

x
x =
x

, x 0
.
, x < 0

SYMMETRY PROPERTY AND SHIFTING

We have already been able to determine the types of graphs of functions and nonfunctions. Next, we shall consider the changes or manipulations that can be made
on those graphs. We may use the symmetry property of a graph to know the graph
on the other sides of the axis. We may also shift the graph up and down or left and
right.

Symmetry Property 1:
The graph of the equation y = f ( x) is said to be symmetrical about the y-axis
if the positive and negative values of x result in the same value of y, that is
f ( x) = f ( x) .

TOPIC 1

FUNCTION AND GRAPH W

15

Example 1.3
Based on the symmetry property 1, you may try to sketch and obtain the
symmetry of the two graphs of:
y = x 2 and y = cos x.
Such function f is also said as even function.

Symmetry Property 2:
The graph of the equation y = g(x) is said to be symmetrical about the x-axis if
the positive and negative values of y result in the same value of x, that is
g(y) = g(-y).

Example 1.4
Based on the symmetry property 2, you may try to sketch the graph of the
equation x = y2. Such symmetry property implies that y is not a function of the
variable x.

Symmetry Property 3:
The graph of the equation y = f ( x) is said to be symmetrical about the origin
if any change in the sign of the value of x results in the change of the sign of
the value of y.

16

X TOPIC 1

FUNCTION AND GRAPH

Example 1.5
If f (x) = x3, we obtain f (x) = x3 = f (x).
Graphically, the symmetry property is illustrated in Figure 1.12.

Figure 1.12

Can you recognise the graphs in Figure 1.12?


Next, let us study about the shifting of a graph.
Suppose the graph of the equation y = f ( x) is already known, we can also
visualise the graph of
y = f (x) + c
and
y = f (x) c,
where we assume that c is a positive constant.
For example, the graph of y = x2 + 2 can be obtained by shifting upward an
amount of 2 units the graph of y = x2.
While the graph of y = x2 2 is obtained by shifting downward the graph of y = x2
an amount of 2 units. This is illustrated in Figure 1.13.

TOPIC 1

FUNCTION AND GRAPH W

17

Figure 1.13: Graph of y = x2 shifted upward and downward

From the graph of y = f ( x) we can also obtain the graph of y = f ( x + c) and


y = f ( x c) . For example, the graph of y = (x + 2)2 is the graph of y = x2 shifted
to the left by an amount of 2 units. Study Figure 1.14.

Figure 1.14: Graph of y = x2 shifted to the left and right

SELF-CHECK 1.2
How do you obtain the graph of y = (x 2)2 from the graph of y = x2?

18

X TOPIC 1

FUNCTION AND GRAPH

EXERCISE 1.3
1.

2.

Determine the symmetry property of each of the following


graph.

(a)

(b)

(c)

(d)

Start with the graph of y = x2, sketch the graph of y = (x 2)


|x 2| + 1.

In topic 1, we have learned about function and the types of function graphs.
From the graph of a function, we are able to determine the symmetry property
and the shifting of a function. We may summarise as follows.

Function
(i)

2
y = x +1
3

One to one relation

(ii)

x2 = y

Many to many relation

TOPIC 1

FUNCTION AND GRAPH W

19

Non Function
(i)
(ii)

x2 + y2 = 25 (Circle)
2

y = x (Parabola)

One to many relation


One to many relation

Symmetry Property 1
f (x) = f (x)

y = f ( x) is said to be symmetrical about the


y-axis if the positive and negative values of x result
in the same value of y.

Symmetry Property 2
g (y) = g (y)

y = g (x) is said to be symmetrical about the xaxis if the positive and negative values of y result in
the same value of x.

y = f ( x) is said to be symmetrical about the


Symmetry Property 3
f (x) = y, f (x) = y origin if any change in the sign of the value of x
result in the change of the sign of the value of y.

Shifting
y = x2 + 2

Shifting upward

y = x2 2

Shifting downward

y = (x + 2)2

Shifting to the left

y = (x 2)2

Shifting to the right

Topic X Limit

LEARNING OUTCOMES
By the end of this topic, you should be able to:

1.

Identify the existence or non-existence of limit at a given point;

2.

Determine the value of a limit using approximation and definition


proving; and

3.

Calculate the limit using the rules of calculating limit.

INTRODUCTION

Any living object will grow at different stages and this can be illustrated as a
function. However, there are several environmental factors that govern the growth
and the continuity of life to happen at a certain limit. This is what is meant by the
limit.
By considering the limit in a function, we need to determine an approximate value
of a function as x approaches to a certain constant. The concept of limit is an
important element in calculus. As it is an important concept, we shall first
approach it informally through the geometrical approach and as an approximation
of the function value at several points close to the point under consideration.
Formally, the definition is used in the proof of the limiting value.
Next, the topic of infinite limit is also discussed so as to complement the
discussion on limit covered in the topic.

TOPIC 2

2.1

LIMIT W

21

LIMIT THROUGH THE GRAPH OF A


FUNCTION

As we have been able to determine the function and several types of graphs of
functions, we are now interested to understand the meaning of the notation

lim f ( x)
xa

where f denotes a function and a may be a real number or an infinity .

lim f ( x) is read as the limit of a function f as x tends to a.


x a

Notice that the notation x a represents the phrase as x tends to a.


Example 2.1
The lim 3 x 2 is read as the limit of the function f (x) = 3x2 as x tends to 1. It
x 1

should be noted that x tends to 1 can either be from the left (x < 1) or from the
right (x > 1).

Example 2.2
The lim 3 x 2 is read as the limit of the function f (x) = 3x2 as x tends to infinitely
x 1

large.

ACTIVITY 2.1
Try to give a similar explanation as above the case of
lim 3 x 2 .
x

Next, we shall learn about the existence or non-existence of a limit.

22

X TOPIC 2

LIMIT

Criteria of the Existence of a Limit


Consider Figure 2.1 and concentrate on x close to 2.

Figure 2.1

In all the three cases above, we can write lim f ( x) = 1 , which means that the
x 2

limits of the three functions as x tends to 2 exist and each is of value 1. Here the
value 1 is said as the value of the limit (as x tends to 2). In other words, we expect
the value of the function f approaches to 1 as x is very close to 2.
From our discussion of the existence of lim f ( x) we have been able to make
x a

several conclusions:
(a)

we do not take particular interest in the value of the function at


x = a, that is f (a) does not affect the existence of lim f ( x) ;
x a

(b)

the function f may not be even defined at x = a; and

(c)

we are only interested with x that is closed to a, not x equals to a!

TOPIC 2

LIMIT W

23

Criteria of the Non-existence of a Limit


If the graph of a function has a jump at a certain point, then the function is said
as not to have a limit at that point.
Example 2.3
Consider Figure 2.2. The graph of the function

f ( x) =

1, x > 0
=
x 1, x < 0
x

has a jump at x = 0. Hence


lim
x a

x
x

does not exist.

Figure 2.2

Figure 2.3 shows a graph that oscillates fast as x approaches to 0.

Figure 2.3

24

X TOPIC 2

LIMIT

Try to sketch the graph of f ( x) = sin

for x close to 0. You will surely obtain the


x

graph as shown in Figure 2.3.

Example 2.4
If the graph of a function is oscillating about a certain point, then the function
does not have a limit at that point. In this case

lim sin
x 0

does not exist.


x

Example 2.5
1
. As x
(x a2 )
tends to a (either from the left or from the right), the value of f (x) is increasingly
large, that is, it is increasing towards an infinitely large value as x approaches to a.
In such cases, we write
Consider Figure 2.4. Observe the graph of the function f ( x) =

lim f ( x) = +
x a

Figure 2.4

ACTIVITY 2.2
Try to obtain the lim f ( x) and lim f ( x) from the respective graphs
x +

of f (x).

Remark: Such function of f is of the class of functions that do not have limit at x =
a.

TOPIC 2

LIMIT W

25

Try the following exercise and you will have a better understanding about the
concept of limit.

EXERCISE 2.1
The graph of a function f is as shown in the following figure.

Determine an integer a, such that lim f ( x) exists, either as a real


x a

number or as an infinity.

2.2

LIMIT THROUGH AN APPROXIMATE


VALUE

We may be given a function without its graph. The problem is how to determine
its limit. To obtain

lim f ( x) ,
x a

we need to substitute several values of x which are closed to a in the function f (x).
Next, by observing the values of f (x) we guess the required limit.

26

X TOPIC 2

LIMIT

Example 2.6
f ( x) =

(a)

1
x

Obtain lim

x +

1
.
x

We substitute several values of x that are closed to an infinitely large


number, such as
x = 1000, f (1000)

= 0.001

x = 106,

f (106)

= 0.000001 = 110-6

x = 1010,

f (1010)

= 110-10.

Thus, we make the following guess

1
= 0.
x + x
lim

Here, we expect that

(b)

1
tends to 0 as x tends to an infinitely large number.
x

1
Obtain lim .
x 1 x
We substitute several values of x that are closed to 1. For example, we use
x = 1.2, 1.01 and 1.0001 as x approaches to 1 from the right and we use
x = 0.8, 0.9 and 0.999 as x approaches to 1 from the left. Note the values of f
(x) in Table 2.1.
Table 2.1

TOPIC 2

LIMIT W

27

We observe that the values of f (x) tend to 1 as x tends to 1, either from the left or
from the right.
The best guess that we can make is

lim
x 1

1
= 1.
x

EXERCISE 2.2
By evaluating the function at certain points, guess the limit of the
following:
(a)

lim

x3 1
x 1

x = 1.1, 1.01, 1.001, 0.9, 0.99, 0.999

(b)

lim

x +1
x3 + 1

x = 1.5, 1.01, 1.001

(c)

lim

x +1
x3 1

x = 0.5, 0.99, 0.999

(d)

lim

x = 0.2, 0.01, 0.0001

x 1

x 1

x 1

x 0

x +1 1
x

We have seen examples to illustrate the calculation of limit exactly and through
approximation method.

28

X TOPIC 2

LIMIT

Example 2.7
(i)

sin x
.
x 0
x

Obtain the value of lim

Table 2.2

Use calculator to complete the above table.

sin x
sin x
= 1 even though
is undefined at x = 0.
x 0
x
x

It seems that lim

Remark: The unit of x used in the calculation above is in radian.


Why is it that the technique of approximation in calculating the limit is not
very convincing? We have even used the word guess! Perhaps it is because
of our own weaknesses. For example, we have chosen an inappropriate
approximate value.
(ii)

sin
.
x 0
x
(It has been stated that this function oscillates very fast as x approaches to 0
so as the limit does not exist).
Obtain the value of lim

When x is substituted by a number close to 0, say x = 0.1, 0.001 and


0.00001 the result is a value of f (x) tat is equal to 0. Hence, we expect that

lim
x 0

sin
= 0.
x

This guess is inexact since in actual fact the limit does not exist.

TOPIC 2

LIMIT W

29

Our weakness in the choice of the values of x as above, each has resulted in

the value of , to be 10, 1000 and 100000 such that sin = 0 .


x
x
Therefore, we have to be very smart in the choice of the values of x. At times, we
make a misleading calculation that is difficult to track.
Now try the following exercises.

EXERCISE 2.3
1.

By evaluating the function at certain points, guess the value of


sin 4 x
lim
; x = +0.2, +0.01, +0.001 .
x 0
x

2.

Given f ( x) =
(a)

x sin x
x3

By evaluating f (x) at x = 0.1, 0.01 and 0.001, guess


lim+ f ( x) .
x 0

(b)

By evaluating f (x) at x = 1 105, 1 106 and 1 107,


guess lim+ f ( x) .
x 0

(c)

2.3

What is the problem raised by the two answers above?

LIMIT THROUGH DEFINITION

We have roughly understood the meaning of the notation lim f ( x) = A .


x a

In general, it is sufficient to state as x tends to a, f (x) tends to A. Remember that


a and A can be real numbers and also be infinite.
We have used two techniques to determine the value of A; through the graph and
approximate value calculation. However, we have not been able to guarantee that
the value of A is the true or false value of the limit.

30

X TOPIC 2

LIMIT

By right, the notation used implies the following:

lim f ( x) = A
x a

The difference between f (x) and A can be made as small as possible by the
choice of x that is sufficiently close to a, except possibly x = a.

Example 2.8
By observing Figure 2.5a, we obtain lim f ( x) = 3 .
x 2

Figure 2.5a

In this case we may made the difference between f (x) and 3 less than 1 by taking
x that is sufficiently close to 2, that is by taking x in the interval (1.5, 2.5).
If we want to make the difference between f (x) and 3 less than 0.5, we may take x
in the interval (1.8, 2.2).

TOPIC 2

LIMIT W

31

Observe Figure 2.5b.

Figure 2.5b

Similarly, we can made the difference between f (x) and 3 as small as possible by
taking x that is sufficiently close to 2, except possibly x = 2.

Example 2.9
h(x) is given in Figure 2.6. In this case,

lim h( x) = 3 .
x 2

Observe that
h( x) 3 < 0.5 as 0 < x 2 < 0.2 .

and
h( x) 3 < 0.3 as 0 < x 2 < 0.1 .

32

X TOPIC 2

LIMIT

Figure 2.6

Now we state the definition of a limit.

Definition 1
Suppose f is a function defined at points about a (except possibly at a) and A is
a real number. The statement lim f ( x) = A
xa

means that for any number > 0 there exists a number > 0 such that
| f (x) A | < as 0 < | x a | < ,
written symbolically as

> 0 > 0 0 < |xa| < | f (x) A| < .

TOPIC 2

LIMIT W

Observe Figure 2.7.

Figure 2.7

EXERCISE 2.4
Determine the largest interval that contains 4 such that the difference
between 3x 7 and 5 is less than:
(a)

0.3

(b)

0.09

(c)

0.00012.

33

34

X TOPIC 2

LIMIT

ACTIVITY 2.3
Obtain more information and try the exercises and examples at the
following website:
http://www.hyper.ad.com/tutor/math/limit_defs.html

Based on the Definition 1, we shall prove the following examples:


(i)
(ii)

lim (3x 7) = 13 and

x 2

sin x
= 1.
x 0
x

lim

Proof (i)

lim (3x 7) = 13 .

x 2

Now, given > 0 we need to choose > 0 such that

3N
x 7
13 <
N


f ( x) A


0 < x
N2 < .
a

as

We need to find (in terms of ) such that |3x + 6| < as 0 < |x + 2| < .
Since

|3x + 6| = |3(x + 2) | = 3| x + 2|

and

3 x + 2 < as x + 2 < .
3

Therefore, we choose = .
3
Thus, given an arbitrary > 0 we choose =

such that

|(3x 7) (13)| < as 0 < |x (2)| < .

TOPIC 2

LIMIT W

35

Hence, it is shown that

lim (3x 7) = 13 .

x 2

EXERCISE 2.5
With reference to the example above, can you prove that
lim(mx + c) = ma + c ?
x a

Proof (ii)

sin x
= 1.
x 0
x

lim

Suppose that x is an acute angle and is measured in radians. Consider the sector of
a circle of radius 1 unit as in Figure 2.8.

Figure 2.8

Area of the triangle OAB Area of the sector OAC Area of the triangle OAC

tan x x sin x

.
2
2
2

36

X TOPIC 2

LIMIT

2
results in
sin x

Multiplying by

x
1

1.
cos x sin x
Inverting,

cos x

sin x
1.
x

This inequality will always be true even if x < 0 since


cos x = cos(x) and

sin( x) sin x sin x


=
=
.
x
x
x

By using the Sandwich Theorem on the inequality

cos x

sin x
1,
x

we obtain

lim

lim 0

sin x
= 1.
x

EXERCISE 2.6
Prove that
(a)
(b)
(c)

x
= 2,
lim 8 4
lim

lim 4 x = 12,

lim 3

lim (3x 7) = 2.

lim 3

We state without proof the Sandwich Theorem as follows.

TOPIC 2

LIMIT W

37

The Sandwich Theorem


Suppose that h(x) f (x) g(x) for each x about the point a, except possibly at
x = a and suppose that

lim h( x) = A = lim g ( x).


x a

x a

Then lim h( x) exists and has a value A.


x a

In the case of

cos x

sin x
1,
x

the graph of the function

f ( x)

sin x
, x0
x

is sandwiched by the graphs of h(x) = cos x and g(x) = 1 (except at x = 0 where f


is undefined). Observe Figure 2.9.

Figure 2.9

Since

lim cos x = 1 = lim1.


x 0

x 0

sin x
= 1.
lim 0
x

The Sandwich Theorem guarantees the lim

38

X TOPIC 2

2.4

LIMIT

RULES OF CALCULATING LIMIT

We have known several limits of functions, such as

lim(mx + c) = ma + c.
x a

This means that

lim c = c

(by taking m = 0)

lim x = a

(by taking m = 1 and c = 0).

x a

and
xa

Beside these two results, the following rules are important.

Rules of Calculating Limit


Suppose that lim f ( x) = A and lim g ( x) = B . Then
xa

(i)

x a

lim( f ( x) + g ( x)) = lim f ( x) + lim g ( x) = A + B;


x a

x a

x a

(Limit of the summation is the summation of limit)


(ii)

lim( f ( x) g ( x)) = lim f ( x) lim g ( x) = A B;


x a

xa

xa

(Limit of the difference is the difference of limit)


(iii) lim( f ( x).g ( x)) = lim f ( x).lim g ( x) = A.B;
x a

x a

x a

(Limit of the multiplication is the multiplication of limit)

f ( x) A
f ( x) lim
x a
(iv) lim
=
= , provided B 0 ; and

xa
g ( x) B
g ( x) lim
x a
(Limit of the division is the division of limit, provided the denominator is
not zero)
(v)

lim n f ( x) = n lim f ( x) = n A ; A 0 for n even.


x a

x a

(Limit of the nth. root is the nth. root of the limit)

TOPIC 2

LIMIT W

39

The above rules given for two functions f and g are still applicable for a finite
number of functions.

Application of Rule (iii) of Calculating Limit


lim( f ( x).g ( x)) = lim f ( x).lim g ( x) = A.B;
x a

x a

x a

(Limit of the multiplication is the multiplication of limit)

Example 2.10
Suppose given,

lim( f1 ( x). f 2 ( x)... f n ( x)) = lim f1 ( x).lim f 2 ( x )...lim f n ( x ).


x a

x a

xa

x a

In particular, if f1 , f 2 ,..., f n are the same function, then we obtain

lim ( f1 ( x) ) = lim f ( x) .
n

xa

x a

(limit of the nth. power is the nth. power of the limit)


From the rule (iii) of calculating limit, where f is a constant function, f (x) = k we
obtain
lim ( k .g ( x) ) = k .lim g ( x)
xa

xa

(The constant factor can be factored out of the limit notation)


lim ( f ( x) g ( x) ) = lim f ( x) lim g ( x) = A B
x a

xa

x a

40

X TOPIC 2

LIMIT

Proof of Rule (ii)


Since lim f ( x) = A , this means that given an arbitrary number > 0 there exists a
x a

number 1 > 0
such that

f ( x) A <

provided 0 < x a < 1.

From lim g ( x) = B , for the same , there exists 2 > 0 such that
x a

g ( x) B <

provided 0 < x a < 2 .

By taking = min {1, 2}, then

f ( x) A <

and g ( x) B <

provided 0 < x a < 2 .

Therefore,

( f ( x) g ( x) ) ( A B ) = ( f ( x) A ) + ( B g ( x) )
f ( x) A + B g ( x)
= f ( x) A + g ( x) B

< + = provided 0 < x a < .


2 2
This completes the proof.
Next, let us consider the proof that the limit is unique, that is if

lim f ( x) = L

(i)

lim f ( x) = M

(ii)

x a

and
x a

then L = M .

TOPIC 2

LIMIT W

41

If > 0 , then
from (i) 1 > 0 f ( x) L <

from (ii) 2 > 0 f ( x) M <

provided 0 < x a < 1 and

provided 0 < x a < 2 .

Hence, provided 0 < x a < = min {1 , 2 } , both are true

f ( x) L <

and f ( x) M <

such that

L M = L f ( x) + f ( x) M L f ( x) + f ( x) M

< + =.
2 2
Therefore, for any > 0 we have
L M < .
If L M, then | L M | > 0. Therefore, the special case that = | L M | we obtain
| L M | < | L M |.
This results in contradiction.
Hence, we conclude that L = M, that is the limit is unique.
We have seen the rules of calculating the limits, we shall now consider some
examples of calculating the limits using the rules.

42

X TOPIC 2

LIMIT

Calculation of Limits Using the Rules


Example 2.11

Refer to Rule (ii)

(ii )
lim (3 x 2 4 x) = lim 3x 2 lim 4 x
x 1
x 1
x 1

Refer to Rule (iii)

(iii )
x 2 4 lim x
= 3lim
x 1
x 1

Refer to Rule (iii)

2
(iii )
x 4 lim x
= 3 lim
x 1
x 1

= 3 (1) 4 (1) = 1
2

Example 2.12

Refer to Rule (iv)

x3 + 9
x3 + 9 (iv) lim
x 3
lim
=
x 3
lim x
x

Refer to Rule (v)

x 3

(v )
=
(i ) 1
=
3

lim ( x3 + 9)
x 3

Refer to Rule (i)

lim ( x3 ) + lim ( 9)
x 3

x 3

3
(iii ) 1
lim x + 9
=
x 3
3
1
27 + 9
=
3
=2

Refer to Rule (iii)

TOPIC 2

LIMIT W

EXERCISE 2.7
Calculate the limit of the followings.
(a)

lim ( 3 x 2 x 3 ) ,

(b)

lim

(c)

lim

x +1 2
,
x 3

(d)

lim

3+ x 3
,
x

(e)

4 x
.
x 16 x 16

x 4

x 8

x 3

x 0

3( x + 1)
,
x4

lim

Example 2.13

lim ( x + 1)
x 3

2 3

Refer to Rule (iii)

x 2 1

) (iii= ) lim ( x +1) lim x 1


(iii )
= ( lim ( x + 1) ) lim x 1
(v )
= ( lim ( x + 1) ) lim x 1
2

x 3

x 3

x 3

x 3

= ( 4)
= 32

2 3

x 3

x 3

Refer to Rule (iii)

Refer to Rule (v)

43

44

X TOPIC 2

LIMIT

For the following examples we use the rules:


lim sin x = sin a,
x a

lim cos x = cos a,


x a

sin x
= 1,
x
tan x
= 1.
lim
x 0
x
lim
x 0

Example 2.14
Since

sin 2 x 2sin 2 x
.
=
2x
x

Therefore,

sin 2 x
2 sin 2 x
= lim
x 0
x 0
2x
x
(iii )
sin 2 x
= 2 lim
x 0
2x
= 2 (1)
= 2.

lim

Example 2.15
sin 3 x sin 3 x
1
=
sin 7 x
x sin 7 x
x
sin 3 x
1
=3
.
3 x 7 sin 7 x
7x

Refer to Rule (iii)

TOPIC 2

Therefore,

sin 3 x
lim

x 0 sin 7 x

sin 3 x

7
= lim 3
x 0
7 sin 7 x
3x

7x

1
sin 3 x
lim
= lim 3

x 0
3 x x0 7 sin 7 x
7x
1
1
3
sin 3 x
= 3lim
= 3(1)
= .

x 0
7(1) 7
3 x 7 lim sin 7 x

x 0
7x

Example 2.16

tan x sin x 1
.
=
x
x cos x
Therefore,

tan x
sin x 1
lim
= lim

x 0
x x 0 x cos x
sin x
1
= lim
lim

x x 0 cos x
x 0
= (1)(1) =1.

LIMIT W

45

46

X TOPIC 2

LIMIT

EXERCISE 2.8
1.

2.

2.5

Calculate the limit of the following.


(a)

lim

(b)

lim

(c)

lim

x 0

tan 2 x
,
3x

sin 2 x
,
x 0
5x
sin 2 x
.
x 0 sin 5 x

Use the Sandwich Theorem to determine:


(a)

1
lim x 2 sin ,
x 0
x

(b)

1
lim x cos .
x 0
x

INFINITE LIMIT

In sections 2.1 and 2.2 we have studied how to obtain the limit of a function
graphically. For example,
lim+

xa

1
1
= + and lim
= +.
2
2
xa ( x a)
( x a)

Please see Figure 2.4.


In general,

lim f ( x) = +
x a

states that the value of f (x) increases towards an infinitely large value as x tends to
a.

TOPIC 2

LIMIT W

47

Example 2.17
x2
x2

x2
1
= lim 2
=
=1
2
x x + 1
x x
1 1+ 0
+
x2 x2

(a)

lim

(b)

x3 4
+ 3
3
x3 + 4
1+ 0
lim 2
= lim x 2 x =
=
x x + 1
x x
1
0
+
x3 x3

(c)

x2 3x
3
3
x 2 3x
0
x
= lim 2 x
= =0
lim 3
x x + 2 x + 6
x x
2x 6 1
+ 3+
3
x
x
x

Definition 2
We write
(i)

lim f ( x) = +
x a

as for every M > 0 there exists a > 0 such that f (x) > M provided 0 < | x a |
< .
(ii)

lim f ( x) =
xa

as for every M < 0 there exists a > 0 such that f (x) < M provided 0 < | x a |
< .

Now, observe how, for example the Definition 2(i) above can be obtained from
the implication of lim f ( x) = A where A is a real number.
xa

The statement we can make the difference between f (x) and A arbitrarily small
should be read as we can make the difference between f (x) and an infinitely large
value A arbitrarily small.

48

X TOPIC 2

LIMIT

The last statement shows that the value of f (x) is very large such that if M is a
positive number, the value of f (x) is larger than M. Please refer Figure 2.10a.

Figure 2.10a

Figure 2.10b

Please see Figure 2.10b as comparison when f (x) < M provided 0 < x a < . We
shall study the proof in the following example.

Example 2.18
1
= +, we have to show that given any number M > 0,
( x a)2
we can obtain a > 0 such that
To prove that lim
xa

1
> M provided 0 < x a < ,
( x a)2
or alternatively

( x a)2 <

1
provided 0 < x a < ,
M

TOPIC 2

LIMIT W

49

but

( x a)2 <

1
1
provided 0 < x a <
.
M
M

Therefore, by choosing =

1
, we obtain
M

1
> M provided 0 < x a < .
( x a)2
This completes the proof.
Now we consider to problem that involves the calculation of limit of the form

lim f ( x) = L and lim f ( x) = L.

x +

In sections 2.1 and 2.2 we have studied how to calculate the limit graphically and
as approximate value. We have known that

lim

1
= 0 and lim e x = 0.
x +
x

In general,

lim f ( x) = L,

x +

states that the value of the difference between f (x) and L can be made arbitrarily
small by taking x to be sufficiently close to an infinitely large value.
The phrase x to be sufficiently close to an infinitely large value means that there
exists a large real number N but x is larger than N.

50

X TOPIC 2

LIMIT

Definition 3
We write
(i)

lim f ( x) = L

x +

if for every > 0 there exists N > 0 such that


| f (x) L | < provided x > N.
(ii)

lim f ( x) = L

if for every > 0 there exists N > 0 such that


| f (x) L | < provided x > N.

Study Figure 2.11 closely.

Figure 2.11

Example 2.19
We use the Definition 3(i) to show that lim

x +

1
= 0.
x

Here we need to find a number N > 0 such that

1
0 < provided x > N .
x

TOPIC 2

Observe that
1
1 1
0 = = for x sufficiently large.
x
x x
Furthermore,

1
1
< provided x > .
x

Therefore, by taking N =

, we obtain

1
0 < provided x > N .
x
1
= 0.
x + x
lim

This shows that

Please try the following exercises.

EXERCISE 2.9
l.

Determine the limit of the following


(a)

lim

x2 + 1
x
x2

(d)

lim

x2 1
x x 3 + 1

(b)

lim

x2
x x 2 + 1

(e)

lim

(c)

lim

3x 2
x x 2 + 1

(f)

lim

x3 + 1
x x 2 1

x3 + 2 x 2 + 4
x x 3 + x 2 + 8

LIMIT W

51

52

X TOPIC 2

2.

LIMIT

Given the positive numbers and L, the limit of the function


f (x) at . Determine the negative number N such that

f ( x) L < provided x < N.

3.

(a)

lim

1
= 0;
x x 2

= 0.001,

(b)

x +1
= 1;
x
x

= 0.01.

lim

Use the definition to prove the following.


(a)

(b)

lim

1
= 0,
x

x
= 1,
x + x + 1
lim

1
= +,
x

(c)

lim

(d)

1
lim 4 = .
x 0
x

x 0

In this topic we have learnt

2 methods of determining and approximating the value of limit, namely the


graphical method and the approximate value method.

We have used the definition to prove the limit.

In this topic we have also been shown how to use the rules of calculating the
limit of several functions.

We have been introduced the topic of infinite limit.

T o p ic

Continuity

LEARNING OUTCOMES
By the end of this topic, you should be able to:
1.

Prove the continuity of a function at a given point;

2.

Rewrite the conditions of continuity of a function; and

3.

Apply the definition of limit to determine the point of continuity of


a function.

INTRODUCTION

This topic explains the importance of the value of a function in order to determine
the continuity of the function at given points. A function, which is undefined at a
given point, is definitely not continuous at that point. The properties of a
continuous function and The Mid Value Theorem are also explained in details.

3.1

THE MEANING AND PROPERTIES OF A


CONTINUOUS FUNCTION

Your knowledge and understanding of function and graph of a function form the
basis in the study of continuity of a function. In the topic of continuity, the value
of a function at the limiting point is of very importance. If a function is undefined
at a point say x = a, then it is definitely not continuous at that point.

54

X TOPIC 3

CONTINUITY

Non continuous function


F(a) Undefined function
Example 3.1
f (x) = 1, x 2 is not continuous at the point x =2 and
g (x) = x 1, x 3 is not continuous at the point x = 3.
Example 3.2

h( x ) =

x2 1
, x 1 is not continuous at x = 1.
x +1

Other than that, the value of a function at the point also plays a very
important role. If the value of the limit at x = a is not the same as the value
of the function at x = a, which is f (a) then the function is not continuous at
x = a.
(b)

The Limit of a Function at x = a is Not the Same as the Value of f (a)


Example 3.3
1, x 2

f ( x) = 3
2 , x = 2

is not continuous at x = 2 since the limiting value is 1, that is lim f ( x) = 1


x 2

while the value of the function at the limiting point, f (2) =

3
.
2

Example 3.4

x 1,
g ( x) =
1 ,

x3
x=3

is not continuous at x = 3. In this case, the limiting value is 2 while the


function value is 1, that is lim g ( x) = lim( x 1) = 2
x 3

x 3

while g (3) = 1. In short, lim g ( x) g (3)


x 3

TOPIC 3

(c)

CONTINUITY W

55

lim f ( x) Does Not Exist


x a

Example 3.5

Figure 3.1

Now we consider the graph of the function f as given in Figure 3.1. We


concentrate on the limiting value and the function value at x = 2, 1, 0, 1, 2,
3 and 4.
(i)

The first condition that a function f is continuous at x = a is lim f ( x)


x a

exists.
Therefore, at the point a = 1, 1 and 2, lim f ( x) exists and all are
x a

equal to 2, that is

lim f ( x) = 2, lim f ( x) = 2 and lim f ( x) = 2

x 1

x 1

x 2

At the points a = 2, 0, 3 and 4, lim f ( x) does not exist.


x a

Therefore,
f may be continuous at a = 1, 1 and 2.
At the points a = 2, 0, 3 and 4 the function f is not
continuous.

56

X TOPIC 3

(ii)

CONTINUITY

The second condition that a function f is continuous at x = a is that f


is defined at x = a.
Since f is undefined at a = 2, therefore f is not continuous at a = 2.
Now, we are left with the points a = 1 and 1.

(iii) The third condition that a function f is continuous at x = a is that the


function value and the limiting value are the same at x = a, that is
lim f ( x) = f (a ) .
x a

Now, we refer to lim f ( x) = f (a) .


x a

At the point a = 1, the third condition is satisfied since lim f ( x) = 2


x 1

and f (1) = 2.
At the point a = 1, the third condition is not satisfied since
lim f ( x) = 2 the function, f (1) = 3.
x 1

Therefore, f is not continuous at a = 1.


Hence, we conclude that the function f in which the graph is given in Figure 3.1,
is not continuous at the points a = 2, 0, 1, 2, 3 and 4.
At the point a = 1 the function f is continuous. In actual fact, the function f is
continuous at every point in the interval [2, 4] except at the points a = 2, 0, 1, 2,
3 and 4. As a conclusion from the given examples, we observe that there are 3
conditions to be fulfilled for a function to be continuous.

TOPIC 3

CONTINUITY W

57

Definition 4
A function f is said to be continuous at x = a provided the following 3
conditions are satisfied:
1.

f is defined at x = a, that is f (a) exists;

2.

lim f ( x) exists; and

3.

lim f ( x) = f (a ) .

x a

x a

If any one of the conditions is not satisfied, then f is not continuous at the point
x = a.

Figure 3.2

Figure 3.2a shows that f is not defined at x = a.


Figure 3.2b and Figure 3.2c show that lim f ( x) does not exists.
x a

Figure 3.2d shows the limiting value and the function value are not the same. In
actual fact, the conditions 1 and 2 of Definition 5 above are already implicit in
condition 3. However, conditions 1 and 2 are included to further clarify the
definition. In any proof of continuity it is better to show that condition 3 is also
satisfied.
Please see examples in the following website:
http://archives.nath.utk.edu/visual.calculus/1/continuous.5

58

X TOPIC 3

CONTINUITY

Example 3.6
Determine whether the following functions are continuous at x = 3.
x2 9
;
x 3

(i)

f ( x) =

(ii)

x2 9
, x3

g ( x) = x 3
4 , x=3

x2 9
,

(iii) h( x) = x 3
6 ,

x3
x=3

Solution:
(i)

The function f is undefined at x = 3. Therefore, lim f ( x) f (3) .


x 3

Hence, f is not continuous at x = 3.


(ii)

The value of g at x = 3 is 4, that is g (3) = 4, but


x2 9
( x 3)( x + 3)
= lim
x 3 x 3
x 3
x 3
= lim ( x + 3) = 6,

lim g ( x) = lim
x 3

x 3

that is, lim g ( x) g (3) . Therefore, g is not continuous at x = 3.


x 3

(iii) The function h is continuous at x = 3 since


x2 9
=6
h (3) = 6 and lim h( x) = lim
x 3
x 3 x 3
that is lim h( x) = h(3).
x 3

TOPIC 3

CONTINUITY W

59

Continuity in an Interval
Next, a function is said to be continuous in an open interval provided it is
continuous at every point in the interval.
With reference to example 3.6, the functions f and g above are both continuous in
the interval (, 3) (3, +) while the function h is continuous in the interval
(-, +). The functions f and g above are both continuous at every real number
except at x = 3; while the function h is continuous at every real number. The
function h which is continuous at every real number is also said to be continuous
everywhere.
Whenever a function, say p, is continuous in an open interval (c,d) and
furthermore
lim p ( x) = p (d ), lim+ p ( x) = p (c)

x d

x c

the function is said to be continuous in the closed interval [c,d].


Note: Geometrically, a function is continuous in an interval if its graph can be
traced without lifting the pencil from the paper. Please note that this is not the
definition of continuity but only a physical interpretation.
Thus discontinuity happens if the graph has a break. For example, an electric
cable is broken at t = t0, and then the voltage will immediately drop to zero. Figure
3.3 gives the graph of voltage versus time and discontinuity at t = t0.

Figure 3.3

60

X TOPIC 3

CONTINUITY

Example 3.7
Now consider the technique of showing that the function

1
x sin , x 0
f ( x) =
x

0
, x=0

is continuous at x = 0.
It is clear that f is defined at x = 0 and f has a value 0, that is f (0) = 0. Next,
show that the value of the limit is also 0.

1
lim f ( x) = lim x sin = 0
x 0
x 0
x

Therefore, f is continuous at x = 0.
We shall note several properties of a continuous function that may simplify our
study of continuity.
Property A
If k is a real number, f and g are continuous at x = a, then the following functions
are continuous at a.
kf , f + g , f g , f .g , f n ,
(provided g (a) 0 ) and

f
g

f (provided f (a) > 0 whenever n is even).

Property B
The polynomial function, the rational function, the power root function and the
trigonometric function are all continuous at every point in their respective
domains.
Property C
If g is continuous at a and f is continuous at g (a), then f o g is continuous at a.
Note the application of Property A in the following example.

TOPIC 3

CONTINUITY W

61

Example 3.8
7 x 5 35 x + 6
.
x 2
3x3 8

Determine lim

Here, g(x) = 3x3 8 and g(2) = 16 0. Therefore,


7 x 5 35 x + 6 7(2)5 35(2) + 6 160
=
=
= 10.
x 2
3x3 8
16
16

lim
Example 3.9
Determine lim
x 2

x 2 + 3 x 10
.
x2

Here, x = 2 is not included in the domain of f ( x) =

x 2 + 3 x 10
. Therefore, the
x2

Property A cannot be used. However, by factorising,


lim
x 2

Remark:

(i)

(ii)

x 2 + 3 x 10
( x 2)( x + 5)
= lim
= lim( x + 5) = 7.
x 2
x 2
x2
x2

The second equality is obtained since in this case, x 2 such


x2
that
= 1.
x2
( x 2)( x + 5)
The function f ( x) =
is not the same as the
( x 2)
function m(x) = x + 5.

You may see examples for Property B and C in the website.


Please visit the following website:
http://archives.math.utk.edu/visual.calculus/1/continuous.5/

62

X TOPIC 3

CONTINUITY

Please test your understanding with these exercises.


EXERCISE 3.1
l.

2.

3.2

Determine the point at which the following functions are not


continuous.
(a)

f ( x) =

x
x +1

(b)

f ( x) =

x
x 1

(b)

f ( x) =

x2
sin x

(d)

f ( x) =

x2 4
x3 8

Obtain the value of k such that the following functions are


continuous.
(a)

kx 2 , x 2
f ( x) =
2 x + k , x > 2

(b)

2
1
x cos , x 0
f ( x) =
x

k
, x=0

MID VALUE THEOREM

We have known that the graph of a function continuous at a point is not broken at
that point. Next, a function that is continuous in an interval has its graph
continuous in the interval. Please see Figure 3.4, which gives examples of the
graphs of continuous functions in an interval [a, b].

Figure 3.4

TOPIC 3

CONTINUITY W

63

Based on the Figure 3.4, we state without proof the following theorem.
Mid Value Theorem
Suppose f is a continuous function in a closed interval [a,b] and k is an arbitrary
real number between f (a) and f (b), then there exists c in (a,b) such that f (c) = k.
In other words, a continuous function will attain every value between its extreme
values.
Note how we use the Mid Value Theorem in the following examples.
Example 3.10
After ten minutes from the departure, the velocity of an airplane reaches
500 knots. How could you made a conclusion that several minutes earlier the
airplane velocity has reached 345 knots?
Explanation
The velocity l of an airplane is a continuous function of the variable time.
Specifically, the velocity function is continuous in the time interval [0, 10]. The
velocities at the extremes of the interval are
l(0) = 0 and l(10) = 500.

Since 345 knots is between 0 and 500 knots, and l is continuous in the interval
[0, 10], then by the Mid Value Theorem, there exists c in [0,10] such that
l(c) = 345 knots.

This means that there is an instant that the airplane velocity is 345 knots.
Example 3.11
Show that given any circle, say of radius j, there exists a square whose diagonal is
between j and 2j and the area is half of the area of the circle.
Solution
Let the length of the sides of the square be x. Therefore, the area is x2 and the
length of the diagonal is

2x2 = 2x .

64

X TOPIC 3

CONTINUITY

Note that

j
x
2

j 2x 2j

2 x

j2
x2 2 j 2.
2

The area function of the square f (x) = x2 is continuous in any interval.


j

Specifically, f is continuous in the interval


, 2 j and the values of f at the
2

extremes of the interval are

j j2
f
= 2 and f
2

2 j = 2 j 2.

Since f is continuous in
, 2 j and
2

1
j j2
2
2
f
= 2 2 j 2 j = f
2

2 j

therefore, by virtue of the Mid Value Theorem, there exists c


, 2 j such
2

1
that f (c) = j 2 .
2
The Mid Value Theorem guarantees the existence of at least a number c in the
interval [a, b]. Sometimes, there exists more than one c. See for example,
Figure 3.4c. However, the Mid Value Theorem does not state how to obtain the
number c. Such theorem is known as theorem of existence.

TOPIC 3

CONTINUITY W

65

Try the following exercises to test your understanding of the topic.

EXERCISE 3.2
1.

Use the Mid Value Theorem to guarantee the existence of the


roots of the equation 4x3 6x2 + 3x 2 = 0 in the interval
(1, 2).

2.

If f (x) = x3 x2 + x, show that there exists a number c


such that f (c) = 10.

In this topic we have been exposed to the concept of a continuous function and
its conditions.

The method of calculating limits using the Rules of Calculating Limits and the
Sandwich Theorem are illustrated using simple examples to be understood.

In this topic the usage of the Mid Value Theorem to determine a point in a
specified interval for a given function is illustrated.

T o p ic

Tangent Line
and
Derivative

LEARNING OUTCOMES
By the end of this topic, you should be able to:

1.

Determine the average and instantaneous speeds, given the time


interval;

2.

Determine the value of the slope of a tangent line at a specified


point; and

3.

Determine the derivative of a function at a given point.

INTRODUCTION

In Topic 3 we have learnt about limit and continuity. With the knowledge of limit,
we will be able to determine the derivatives of a function. In Topic 4 we will be
introduced to the concept of speed as the rate of change of displacement with
respect to time and the derivatives of 4 primary functions.
A speed can be either an instantaneous speed or an average speed. By using the
formula for an average speed, we will be able to find an instantaneous speed. An
instantaneous speed can also be defined geometrically as the rate of increment of
the tangent line in terms of the displacement with respect to time. This definition
is used to approximate a speed from a given graph of displacement against time.
The concept of derivative is important in our everyday activities such as in
economics theory where we approximate the expenditure limits, income and
production rates.

TOPIC 4

4.1

TANGENT LINE AND DERIVATIVE W

67

INSTANTANEOUS SPEED AND SLOPE OF


TANGENT LINE

In Topic 3 we have been introduced to the concept of limit where it plays an


important role in the determination of speed. Speed is the rate of change of
displacement of an object. There are two types of speed, namely the average
speed and the instantaneous speed. However, a speed normally refers to an
instantaneous speed.

4.1.1

Average Speed

The displacement of an object after t seconds falling from a building is given by


the function
s(t) = 5t2 meters.
We shall consider a technique of determining the speed of the object at time t = 2.
In the time interval [2,5] seconds, that is from t = 2 to t = 5, the change of
displacement is given by
s = s(5) s(2) = 125 20 = 105 meters.
So the average speed in the interval [2,5] seconds is

s s (5) s (2)
=
= 35m/s.
t
52
By using the same approach, we can obtain the average speed in every interval
that follows. We can also expect that the average speed will not be 20 m/s since
the width of the time interval cannot be zero.
Remark: We are more interested in the instantaneous speed rather than the
average speed in a time interval.

68

X TOPIC 4

TANGENT LINE AND DERIVATIVE

Table 4.1
Time Interval
(seconds)

t
(seconds)

s
(meter)

Average Speed

s
t

(meter/second)
[2,3]

S(3) S(2) = 25

25

[2,2.2]

0.2

S(2.2) S(2) = 4.2

21

[2,2.01]

0.01

S(2.01) S(2) = 0.2005

20.05

[2,2.001

0.001

S(2.001 S(2) = 0.020005

20.005

ACTIVITY 4.1
Can you make a conclusion about the relation between the time
interval and the average speed?

4.1.2

Instantaneous Speed

We shall try to find the instantaneous speed for the function s(t) = 5t2, at t = 2. We
observe the following solution.
In the interval [2,t] seconds, t 2 0.
s s (t ) s (2) 5t 2 20 5(t 2)(t + 2)
The average speed =
=
=
=
.
t
t2
t2
t 2
Since the width of the interval cannot be zero, that is t = 2 0.
Therefore,
average speed =

s
= 5(t + 2).
t

As t tends to 2, the value of 5(t + 2) tends to 20. As we have already learnt in


s
Unit 1, the value of
= 5(t + 2) can be made as close to 20 as possible by taking
t
t that is very close to 2. Symbolically,

TOPIC 4

lim =
t 2

TANGENT LINE AND DERIVATIVE W

69

s
s (t ) s (2)
= lim
= lim 5(t + 2) = 20.
t

2
t 2
t
t2

This is the instantaneous speed at t = 2 which we are trying to obtain.


Definition of Instantaneous Speed
If s = f (t) is a function which gives the displacement of an object in time t then v,
the instantaneous speed of the object at time t = t0 is given by the limit of the
s
average speed
as t approaches to 0. We give the definition of an
t
instantaneous speed as follows:
Definition of Instantaneous Speed
v = lim

t 0

f (t ) f (t0 )
s
= lim
t

t
0
t
t t0

provided the limit exists.

EXERCISE 4.1
A ball is thrown into the air. After t seconds, the height is given by
s = 10t 5t2 meter.
(a)

Calculate the average speed in the intervals [0,1], [1,2] and


[0,2].

(b)

Determine the instantaneous speed at t = 0, t = 1 and t = 2.

Test your understanding by doing the exercises above.


Next, we shall study an instantaneous speed using the graph of a function.

70

X TOPIC 4

4.1.3

TANGENT LINE AND DERIVATIVE

Instantaneous Speed Using Graph of a Function

Example 4.1

Figure 4.1

The graph of the function y = f (x) is given in Figure 4.1.


Note that the straight line, which joins two points on the graph, (c, f (c)) and
(d, f (d)) has a slope

f ( d ) f (c )
.
d c
This straight line is known as the secant line and there are several secant lines that
pass through the point (c, f (c)).

Figure 4.2

TOPIC 4

TANGENT LINE AND DERIVATIVE W

71

With reference to Figure 4.2, as x approaches to c, the point (x, f (x)) moves closer
to the point (c, f (c)) along the graph of y = f (x). The secant line will also move
towards the straight line called the tangent line of f at x = c.
The slope of the secant line will consequently approach the slope of the tangent
line.
Since the slope of the secant line which joins the points (c, f (c)) and (x, f (x)) is
f
f ( x ) f (c )
=
,
xc
x

then the slope of the tangent line is


f
f ( x ) f (c )
= lim
x 0 x
x c
xc
lim

Definition of Slope of Tangent Line

Figure 4.3

With reference to Figure 4.3, the slope of the tangent line of f at x = c is


lim
x c

f ( x ) f (c )
provided that this limit exists.
xc

72

X TOPIC 4

TANGENT LINE AND DERIVATIVE

This limit is usually written as f (c) , that is:


f (c) = lim
xc

f ( x ) f (c )
.
xc

So the equation of the tangent line of f at x = c is


y f (c) = f (c)( x c).

Note: We end this section by emphasising that an average speed is equivalent to


the slope of a secant line while instantaneous speed is equivalent to the
slope of a tangent line.
Note that lim
x c

f ( x ) f (c )
xc

can also be interpreted as an average rate of

change of f with respect to x in the closed interval [c,x]. Consequently,


f (c) = lim

x 0

f
f ( x ) f (c )
as the instantaneous rate of change of f at x = c.
= lim
x

c
x
xc

Try out the following questions to test your understanding of the preceeding
discussion.

EXERCISE 4.2
Consider f (x) = 10x 5x2.
(a)

Determine the slope of the tangent line at c = 0, 1 dan 2.

(b)

Obtain the tangent line that passes through every point in (a)
above.

(c)

Sketch the graph of f and show the respective tangent lines.

TOPIC 4

4.2

TANGENT LINE AND DERIVATIVE W

73

DERIVATIVES

The concept underlying our discussion about an instantaneous speed, slope of a


tangent line and the instantaneous rate of change in Section 4.1 above is the
concept of limit, that is:
lim
x c

f ( x ) f (c )
.
xc

We have also been able to define derivative as follows:

Definition of Derivative
The derivative of a function f at c is given by
lim
x c

f ( x ) f (c )
.
xc

Example 4.2
Find the derivative of f (x) = x2 + 3 at c = 1.
Solution
Consider
f (x) f (c) = (x2 + 3) (1 + 3) = x2 1 = (x + 1)(x 1).
So
lim
x 1

f ( x) f (1)
( x 1)( x + 1)
= lim
= lim( x + 1) = 2.
x

1
x 1
x 1
x 1

Remark: The derivative of f at c is denoted by f (c) . The process of the


determination of a derivative is known as differentiation. If f (c)
exists, the function f is said to be differentiable at c.

74

X TOPIC 4

TANGENT LINE AND DERIVATIVE

Example 4.3
1

Is f ( x) = x 2 = x differentiable at c = 1?

Solution
We determine the existence of f (1) .
lim
x 1

x 1 x +1
f ( x ) f (1)
x 1
= lim
= lim

1
x

1
x 1
x 1
x + 1
x 1
x 1
1
1
= lim
= lim
= .
x 1
x 1
( x 1) x + 1
x +1 2

So f (1) exists and has a value

1
. Therefore, f ( x) =
2

x is differentiable at c = 1.

Remark: Both of the examples 4.2 and 4.3 are solved by using the Definition of
Derivative as method of the first derivative.
Try out the following question.

EXERCISE 4.3
Determine the derivative of the function f (x) = 3 + 2x at c = 1 using
the definition of derivative
f (c) = lim
x c

f ( x ) f (c )
.
xc

By substituting x = c + x in
lim
x c

f ( x ) f (c )
,
xc

we obtain the second from of the derivative of f at c as:


lim

x 0

f (c + x) f (c)
.
x

TOPIC 4

TANGENT LINE AND DERIVATIVE W

75

Example 4.4
Find the differentiation of f ( x) =

2
at c from the list principles.
x

Solution
Note that
f ( c + x ) f ( c ) =

2
2 2c 2(c + x)
2x
=
=
.
c + x c
(c + x)c
(c + x)c

(We assume here that c 0 since 0 cannot be in the domain of f )


So
f (c) = lim

x 0

f ( c + x ) f ( c )
2
2
= lim
= 2 , c 0.
0

x
c
c(c + x)

This second method of obtaining the derivative simplifies the visualisation of f


as a function. The function is derived from f through
f ( x) = lim

x 0

Example 4.4 illustrates that if f ( x) =


f ( x) =

f ( x + x) f ( x)
.
x

2
, then
x

2
2
, provided x 0 (based on f (c) = 2 ) .
2
x
c

ACTIVITY 4.2
Try to find the derivative of g ( x) = x using the second approach
and obtain
f ( x) =

2
, provided x 0.
x2

76

X TOPIC 4

TANGENT LINE AND DERIVATIVE

EXERCISE 4.4
(a)

By using
lim

x 0

f ( x + x ) f ( x )
,
x

find the derivative of f (x) = 3 + 2x at c = 1,

1
and 4.
2

(b)

Based on the result in (a) determine f(x); and

(c)

Determine f ( x) by using
lim

x 0

f ( x + x) f ( x)
.
x

Next, we see geometrically that a function is not differentiable at c.

Example 4.5
Prove that f ( x) = x 1 is not differentiable at 1.
Solution
lim

x 0

1 + x 1
x
f (1 + x) f (1)
= lim
= lim
.
x 0
x 0 x
x
x

Since
x 1 ,
=
x 1 ,

x > 0
x < 0

then
lim

x 0

x
does not exist.
x

x
x

= 1
= 1 while lim
lim
+
x 0 x
x0 x

TOPIC 4

TANGENT LINE AND DERIVATIVE W

77

Therefore, f (x) = | x 1| is not differentiable at 1. Remember that the graph of f


is as given in Figure 4.4, that is there is a sharp turn at 1.

Figure 4.4

Example 4.6
1

Now consider f ( x) = x 3 . Its graph is given in Figure 4.5.

Figure 4.5

78

X TOPIC 4

TANGENT LINE AND DERIVATIVE

Investigate the existence of f at 0.

Solution
1

f (0 + x) f (0)
(x) 3
lim
= lim
= lim
x 0
x 0 x
x 0
x

As we have learnt about continuity in Topic 3, lim

x 0

1
( x )
1

(x)

2
3

2
3

= + means that the

secant line has its slope approaches to an infinitely large value. In such case, that is
lim

x 0

f (0 + x) f (0)
f (x) f (0)
= lim
= +
x 0
x
x

the vertical line x = 0 is a tangent line of f at 0.


1

Hence, the function f ( x) = x 3 is not differentiable at c = 0.


Examples 4.5 and 4.6 illustrate geometrically that a function is not differentiable
at c if:
(a)

there exists a sharp turn at c or the tangent line at c is vertical; and

(b)

the function is not continuous at c as proved in the following theorem, that


is:

If f is differentiable at c then f is continuous at c.

Remark: This theorem is equivalent to the statement: If f is not continuous at c


then f is not differentiable at c.

TOPIC 4

TANGENT LINE AND DERIVATIVE W

Proof
Suppose that f is differentiable at c, that is f' (c) = lim
x c

79

f ( x ) f (c )
exists. We are
xc

interested to show that f is continuous at c, that is:


lim f ( x) = f (c) or lim ( f ( x) f (c) ) = 0.
x c

x c

(Please refer to Section 3.1)


Since

f ( x ) f (c ) =

f ( x ) f (c )
( x c), x c
xc

therefore,
f ( x ) f (c )

lim ( f ( x) f (c ) ) = lim
( x c)
x c
x c
xc

f ( x ) f (c )
= lim
x c)
lim(
x c
xc

x c
= f (c) 0
=0

Remark: The final equality is obtained since f (c) exists (as a real number;
not , for example).
The theorem should not be assumed to be equivalent to the statement: If f is not
differentiable at c, then f is not continuous at c. In actual fact, we have already
seen that f (x) = |x 1| is not differentiable at 1, but f is continuous at 1.
Furthermore, f is continuous at every point in the interval (, +).
We have seen for example g ( x) = x , its derivative is
g' ( x ) =

1
2 x

provided x 0.

This indicates that g is differentiable at every real number x, except x = 0.


Actually, g is differentiable at every x satisfying 0 < x < +.

80

X TOPIC 4

TANGENT LINE AND DERIVATIVE

In such cases, g is said to be differentiable in the interval (0,+). In particular, g is


also differentiable in the interval (1,5), for example.
The derivative of the function y = f (x) has been denoted by f . It can also be
denoted by several notations such as

df d
dy
, ( f ( x) ) , y' ,
and Df (x).
dx dx
dx
Next, by reminding that
f ( x) = lim

x 0

f ( x + x ) f ( x )
,
x

we can consider it as a function. Hence, the function f may be also differentiable


and we can investigate the existence of
g ( x + x) g ( x)
x 0
x
lim

where g = f .

Example 4.7
Find the derivative of f (x) = x2 + 3.
Solution
( x + x) 2 + 3) ( x 2 + 3)
(
f ( x + x ) f ( x )
lim
= lim
x 0
x 0
x
x
x ( 2 x + x )
= lim
x 0
x
= lim ( 2 x + x )
x 0

= 2 x.

Therefore, f (x) = 2x, that is, f is a differentiable function. Furthermore, f itself


is a mapping of x into 2x.

TOPIC 4

TANGENT LINE AND DERIVATIVE W

81

Example 4.8
Suppose g(x) = f (x) = 2x is given. Is g(= f ) differentiable?
Solution
Since
lim

x 0

g ( x + x) g ( x)
2( x + x) 2 x
= lim

0
x
x
2x
= lim
x 0 x
= 2.

Therefore, g(= f ) is differentiable and we write as


g(x) = f (x) = 2.

Remark: f ( x) is known as the second derivative of f and we usually denote it as


d2 f
d2y
,
y
,
and D2f (x).
2
2
dx
dx

The higher derivatives can be similarly determined.

Derivative of a Piecewise Function


Now, we consider the piecewise function such as
2x2 + 4
f ( x) = 2
x + 1

,
,

x<1
x1

We are interested to investigate whether it is differentiable or otherwise.


At the point x > 1, we noticed that

( ( x + x)2 + 1) ( x2 + 1)
f ( x + x) f ( x)
= lim
x 0
x 0
x
x
2
2 x(x) + (x)
= lim
= lim (2 x + x) = 2 x.
x 0
x 0
x

f ( x) = lim

82

X TOPIC 4

TANGENT LINE AND DERIVATIVE

Therefore f, is differentiable at every x greater than 1.


At the point x < 1, we obtained that

( 2( x + x)2 + 4 ) ( 2 x2 + 4 )
f ( x + x) f ( x)
= lim
x 0
x 0
x
x
2
4 x(x) 2(x)
= lim
= lim (4 x + 2x) = 4 x.
x 0
x 0
x

f ( x) = lim

Therefore f, is differentiable at every x less than 1.


Now, we have to determine whether it is also differentiable at x = 1, or otherwise?
So we have to determine
lim

x 0

f (1 + x) f (1)
x

Thus, we need to obtain the limits from the left and right of 1 (if not what value
should f (1 + x) assumed?)
2(1 + x) 2 + 4 ) (12 + 1)
(
f (1 + x) f (1)
lim
= lim
x 0
x 0
x
x
4x + (x) 2
= lim
= lim (4 + x) = 4.
x 0
x 0
x

( (1 + x) + 1) (1 + 1)
f (1 + x) f (1)
= lim+
x 0
x
x
2x + (x) 2
= lim+
= lim+ (2 + x) = 2.
x 0
x 0
x
2

lim

x +0

Since the limit from the left is not the same as the limit from the right, therefore it
is not differentiable at x = 1, that is f (1) does not exist.
Therefore,
2x2 + 4
f ( x) = 2
x + 1

,
,

x<1
x1

TOPIC 4

TANGENT LINE AND DERIVATIVE W

83

we obtained
4x2
f ( x) = 2
2x

,
,

x<1
x>1

4
f ( x) =
2

,
,

x<1
x>1

f ( x) = 0,

x 1.

( f (1) does not exist).


Similarly, we can show that

and

Try out the following question to test your understanding of our preceding
discussion.

EXERCISE 4.5
Show that g(x) = |x| is differentiable at every value of x except x = 0.

ACTIVITY 4.3
You need to perform the activities in the animation and see
the explanation on the tangent line and derivatives.
http://archives.math.utk.edu/visual.calculus/2/tangents.8/index.html

84

X TOPIC 4

4.3

TANGENT LINE AND DERIVATIVE

DERIVATIVES OF SOME PRIMARY


FUNCTIONS

The primary functions in which the derivatives are to be determined are:


(a)

The Constant Function f (x) = k, where k is a constant;

(b)

The Power Function f (x) = xn, where n is a rational number;

(c)

The Exponential Function f (x) = exp(x) = ex; and

(d)

The Trigonometric Function f (x) = sin x.

Let us consider the 4 theorems as the results of those 4 functions above and the
proofs of each of these theorems.

Theorem 1 The Constant Function


The derivative of a constant function f (x) = k is a zero.
Proof
Theorem 2 The Power Function
For an arbitrary rational number n, the function f (x) = xn is differentiable and
d n
x ) = nx n 1.
(
dx
Proof
We prove for the case of n a positive integer greater than 1.
By using the binomial expansion

( x + x )

= x n + nx n 1 ( x ) +

n(n 1) n 2
2
n
x ( x ) + ... + ( x ) ,
2

TOPIC 4

TANGENT LINE AND DERIVATIVE W

85

we obtain

( x + x ) x n
f ( x + x) f ( x)
= lim
x 0
x
x
n

lim

x 0

n(n 1) n 2
2
n
x ( x ) + ... + ( x ) x n
2
= lim
x 0
x
n(n 1) n 2
n 1

x ( x ) + ... + ( x )
= lim nx n 1 +
x 0
2

n 1
= nx + 0 + ... + 0
x n + nx n 1 ( x ) +

= nx n 1.

For the case n = 1, we can easily prove that.


lim

x 0

that is

( x + x ) x = lim x = 1
f ( x + x ) f ( x )
= lim
x 0
x 0 x
x
x

d
( x ) = 1 and this satisfies with nxn1 for n = 1.
dx

We deter the proofs for the cases of n a negative integer and an arbitrary rational
number.
Table 4.2: Examples of Derivatives of Power Function
Function

Derivative

f ( x) = x

f ( x) = 2x

f ( x) = x

f (x) = 5x4

f ( x) = x

1
x3

f ( x) =

f ( x) =

1
3

x2

f ( x) =

d 12 1 12
1
x = x =
dx 2
2 x

f ( x) =

d 3
( x ) = 3x 4 = x34
dx

f ( x) =

d 23
2 5
2
x = x 3 = 5
dx
3

3x 3

86

X TOPIC 4

TANGENT LINE AND DERIVATIVE

Remark: The last three functions above are not differentiable at x = 0, that is
f (0) does not exist.
Theorem 3

The Exponential Function


d x
(e ) = ex .
dx

Proof
n

x
x
e = lim 1 +
, from the definition of e
n
n

2
n

x n(n 1) x
x
= lim 1 + n +
+
+
...

.
n
n
2
n

Now,
e x
1 (n 1)
1
n 1

= lim 1 +
+
x ) + ... + n ( x ) .
(
x n x
2n
n

Next,
e x +x e x
e x 1
= e x lim
x 0
x 0
x
x

1
n 1
n 1
= e x lim 1 + lim
x ) + ... + n ( x )
(
x 0
n

n 2n
lim

= e x (1 + 0 )
= ex .

TOPIC 4

TANGENT LINE AND DERIVATIVE W

EXERCISE 4.6
Find the function in which the derivative is given by the following:
(a)

(4 + x) 2 16
,
x 0
x

(b)

2 2

lim x 3 ,
x 3 x 3

(c)

lim

(d)

lim

(e)

2(5 + x)3 2(5)3


.
x 0
x

lim

2(c + h) 2 2c 2
,
h 0
h

(3 + h) 2 + 2(3 + h) 15
, and
h 0
h
lim

State also the point at which the differentiation is performed.

Theorem 4

The Sine Function


d
( sin x ) = cos x.
dx

Proof
sin( x + x ) sin x
x
x
x

2sin cos x +

2
2

= lim
x 0
x

sin ' ( x) = lim

x 0

87

88

X TOPIC 4

TANGENT LINE AND DERIVATIVE

x
sin
2 lim cos x + x
= lim

x 0
x x 0
2

2
= 1 cos x
= cos x.
Remember that
lim
h 0

sin h
=1
h

and cosine is a continuous function, that is


lim cos( x + h) = cos( x + 0) = cos x.
h 0

Try out the following exercise.

EXERCISE 4.7
By the using the
d
(cos x) = sin x.
dx

first

principle

of

derivative,

we

have

TOPIC 4

TANGENT LINE AND DERIVATIVE W

89

This topic explains:

The relationship between derivative and the slope of a tangent line for a given
function.

A differentiation process is performed in order to obtain the derivative of a


function.

Use the two definitions of derivative and to obtain the derivative of several
primary functions.

Able to determine the differentiability of a function or otherwise from the


graph of a function.

T o p ic

Differentiation
Rule

LEARNING OUTCOMES
By the end of this topic, you should be able to:
1.

Use the appropriate rules to determine the derivatives of the given


functions; and

2.

Use the Implicit Differentiation to determine the tangent line of a


given equation.

INTRODUCTION

In Topic 5, we explain the use of the Differentiation Rules of determining the


derivatives of several given types of functions. We shall also introduce the Chain
Rule and the Implicit Rule of Differentiation.

5.1

GENERAL FORMULA OF DIFFERENTIATION

In Topic 4, we have learned the derivatives of functions through differentiation.


We shall now investigate several special rules of differentiation. Five important
formulas and their simplified form are given in Table 5.1.

TOPIC 5

DIFFERENTIATION RULE W 91

Table 5.1: General Formulas of Differentiation


General Formula of Differentiation

Simplified Form

(i)

d
d
kf ( x ) ) = k ( f ( x ) ) , k is constant
(
dx
dx

(ii)

d
d
d
( f ( x ) + g ( x ) ) = dx
( f ( x ) ) + dx
( g ( x ))
dx

( f + g) = f + g

(iii)

d
d
d
f ( x ) g ( x )) = ( f ( x )) ( g ( x ))
(
dx
dx
dx

( f g) = f g

(iv)

d
d
d
f ( x ) g ( x ) ) = f ( x) ( g ( x ) ) + g ( x) ( f ( x ) )
(
dx
dx
dx

( fg) = fg + f g

(v)

d
d
g ( x) ( f ( x)) f ( x) ( g ( x ))
d f ( x)
dx
dx
,

=
2
dx g ( x )
( g ( x ))

( kf ) = kf

( g ( x)) 0

f gf fg
, g0
=
g2
f

We shall prove 4 of the 5 formulas given, that is we shall give:


Proof (i), Proof (ii), Proof (iv) and Proof (v).
Proof (i)

d
d
kf ( x ) ) = k ( f ( x ) ) , k is constant.
(
dx
dx
By writing F(x) = kf (x), we obtain

d
( kf ( x ) ) = F' ( x)
dx

F ( x + x ) F ( x )
, from the first principle
x 0
x
kf ( x + x) kf ( x)
= lim
x 0
x
f ( x + x) f ( x)
= k lim
x 0
x
d
= k ( f ( x ))
dx

= lim

92

X TOPIC 5

DIFFERENTIATION RULE

Example 5.1
(i)

d 2
d 3
6
4
3 = 2 ( x ) = 2 (3) x = 4 , x 0.
dx x
dx
x

(ii)

d 4 2 4 d 2
4
8
2 j = j.
j = ( j ) =
dj 3
3
3
3 dj

(iii)

d
d
2
( 5x ) = 25 2 ( x 2 ) = 25 2 (2 x) = 50 2 x, is a constant.
dx
dx

Try out the exercise below to test your understanding.


EXERCISE 5.1
By using the general differentiation rule, determine the derivative of
the following function.
f (x) = 3x5

Proof (ii)
d
d
d
f ( x ) + g ( x )) = ( f ( x )) + ( g ( x )) .
(
dx
dx
dx

By writing F(x) = f (x) + g(x), we obtain

d
( f ( x ) + g ( x ) ) = F' ( x)
dx

F ( x + x ) F ( x )
, from the first principle
x 0
x
f ( x + x) + g ( x + x) f ( x) g ( x)
= lim
x 0
x
f ( x + x) f ( x)
g ( x + x ) g ( x )
= lim
+ lim
x 0
x 0
x
x
d
d
= ( f ( x )) + ( g ( x )).
dx
dx

= lim

TOPIC 5

DIFFERENTIATION RULE W 93

Example 5.2
(i)

d 23 d
d 23
x
+
4
5

= 4 x + ( 5)
dx
dx
dx
=4

d 23
x +0
dx

2 13
= 4 x
3
8
= 3 , x 0.
3 x
(ii)

d
d
d 1
sin x + x = ( sin x ) + x 2
dx
dx
dx

= cos x +

1
2 x

, x > 0.

(iii)

d 4x
e 1) = 4e 4 x .
(
dx

(iv)

d 3 x
( e + p ) , p is a constant
dx
d
= 3 (ex )
dx
3 x
= e .

(v)

d
d
e 2 + e 2 x ) = ( e 2 x )
(
dx
dx
= 2e 2 x .

(vi)

d 2
( + sin p + cos x 3e2 x ) , p is a constant
dx
= 0 + 0 sin x 6e 2 x
= sin x 6e 2 x .

94

X TOPIC 5

DIFFERENTIATION RULE

ACTIVITY 5.1
Try on your own to prove the third formula of the general
differentiation rule.

Proof (iv)

d
d
d
f ( x ) g ( x ) ) = f ( x) ( g ( x ) ) + g ( x) ( f ( x ) )
(
dx
dx
dx
By writing F(x) = f (x) . g(x), we obtain
F ( x + x ) F ( x )
d
f ( x ) g ( x ) ) = F' ( x) = lim
, from the first principle
(
x 0
dx
x
but

F ( x + x) F ( x) = f ( x + x) g ( x + x) f ( x) g ( x)
= f ( x + x) g ( x + x) f ( x + x) g ( x) + f ( x + x) g ( x) f ( x) g ( x)
such that

F ( x + x) F ( x)
g ( x + x) g ( x)
f ( x + x) f ( x)
= f ( x + x)
+ g ( x)
.
x
x
x

So
d

g ( x + x) g ( x)
f ( x + x) f ( x)
( f ( x) g ( x) ) = lim
f ( x + x )
g ( x)
+ lim

0
x

0
x
x
dx

g ( x + x ) g ( x )
f ( x + x) f ( x)
= lim { f ( x + x)} lim
{ g ( x)} lim
+ lim

x 0
x 0
x 0
x 0

x
x

d
d
= f ( x) ( g ( x) ) + g ( x) ( f ( x) ) .
dx
dx

Notice that lim f ( x + x) = f ( x). This is because we have assumed that f is


x 0

differentiable and uses the theorem which states that a differentiable function is
continuous.

TOPIC 5

DIFFERENTIATION RULE W 95

Notice also that lim g ( x) = g ( x) since g(x) does not depend on x.


x 0

Example 5.3
(i)

d
dx

(( x

d 3
d
x + 1) + ( x 3 + 1) ( x 2 5 )
(
dx
dx
2
2
3
= ( x 5 ) 3 x + ( x + 1) 2 x

5 )( x 3 + 1) = ( x 2 5 )

= 5 x 4 15 x 2 + 2 x.
(ii)

d
d
3 x 2 sin x ) = 3 ( x 2 sin x )
(
dx
dx
d
d

= 3 x 2 ( sin x ) + sin x ( x 2 )
dx
dx

2
= 3 x cos x + 6 x sin x
= 3 x( x cos x + 2sin x).

Try the exercise below to test your understanding.

EXERCISE 5.2
Using the General Formula of Differentiation, determine the
derivative of the following function
f (x) = (3x 2)(4x + 5).

Proof (v)

d
d
g ( x ) ( f ( x )) f ( x ) ( g ( x ))
d f ( x)
dx
dx
,

=
2
dx g ( x )
( g ( x ))

( g ( x )) 0

96

X TOPIC 5

DIFFERENTIATION RULE

By writing F ( x) =

f ( x)
, we obtain
g ( x)

f ( x + x ) f ( x )

F ( x + x) F ( x) g ( x + x) g ( x) g ( x) f ( x + x) f ( x) g ( x + x)
=
=
x
x
g ( x) g ( x + x) x
g ( x ) f ( x + x ) g ( x ) f ( x ) + g ( x ) f ( x ) f ( x ) g ( x + x )
=
g ( x) g ( x + x) x
=

g ( x) ( f ( x + x ) f ( x) ) f ( x) ( g ( x + x ) g ( x) )

g ( x) g ( x + x) x
f ( x)
g ( x + x ) g ( x )
g ( x)
f ( x + x) f ( x)

.
=

x
g ( x) g ( x + x)
x
g ( x) g ( x + x)
By taking limit when x approaches 0 and using the properties of limit, we obtain
d f ( x)
g ( x) f' ( x) f ( x) g' ( x)

= F' ( x) =
2
2
dx g ( x)
( g ( x) )
( g ( x) )
=

g ( x)

d
d
( f ( x) ) f ( x) ( g ( x) )
dx
dx
.
2
( g ( x) )

Example 5.4
(i)

d
d
( x 3) ( x 2 ) x 2 ( x 3)
d x2
dx
dx

=
2
dx x 3
( x 3)
=

( x 3) 2 x x 2

( x 3)

(ii)
d
d sin x
( tan x ) =
=
dx
dx cos x
=

cos x

x( x 6)
, x 3.
( x 3) 2
d
d
( sin x ) sin x ( cos x )
dx
dx
2
( cos x )

cos 2 x + sin 2 x
= sec 2 x.
cos 2 x

TOPIC 5

DIFFERENTIATION RULE W 97

We have seen the proofs as examples; let us now solve the following exercise.

EXERCISE 5.3
By using the general formulas of differentiation, determine the
derivative of the following function:
1.

2.

x2 + 1
,
3x 2

(a)

f ( x) =

(b)

g (t ) =

(c)

1 t
2
g (t ) =
(1 t ) ,
t

(d)

g (t ) =

cos t
, and
1 sin t

(e)

g (t ) =

1 + tan t
.
1 tan t

1 2t
,
1 + 2t

If g is differentiable, prove that

d 1
g' ( x)
, g ( x) 0.

=
2
dx g ( x)
g
x
( ( ))
By using this formula, show that

d 2
x ) = nx n 1
(
dx

Note: If a falling object follows the path of a curve and not a straight line, then
the chain rule is useful in the determination of the location and time of the
object using the equations of the functions of x and y with respect to time.

98

X TOPIC 5

5.2

DIFFERENTIATION RULE

THE CHAIN RULE

How about the case of the determination of the derivatives of the composite
function? We can use the Chain Rule whereby we exploit our knowledge of the
derivative of a function set. The Chain Rule is a formula that can be used to
determine the derivatives of composite functions. Let us see some examples of the
use of the Chain Rule.

Example 5.5
Find the derivative of y = (3x2 + x 1)5.
Solution:
We can simply expand f (x) = (3x2 + x 1)5 and then obtain the derivative.
However, this approach is a waste of time. Instead, we can use the Chain Rule as
follows:
Step 1
We substitute y = f (u) and u = g(x), to define a composite function
dy
y = ( f D g )( x ) . As we have known,
gives us the rate of change of y with
dx
dy
respect to x. Therefore, the change of y = change of u.
dx
Likewise,
change of u =

du
change of x.
dx

dy du
Therefore, it is obvious that the change of y = change of x.
du dx
This suggests that the rate of change of y with respect to x,

dy dy du
=
dx du dx

TOPIC 5

DIFFERENTIATION RULE W 99

which is equivalent to

d
f ( g ( x ) ) = f ( g ( x ) ) g' ( x ) .
dx

We now state the Chain Rule:

Chain Rule
If y = f (u) is differentiable with respect to u and u = g (x) is differentiable with
respect to x, then y = f ( g ( x ) ) is differentiable with respect to x and
furthermore

dy dy du
=
.
dx du dx

Step 2

dy
du
= 5u 4 and
= 6 x + 1.
du
dx

By writing u = 3x2 + x 1 we obtain y = u2. Hence,


Therefore,

4
dy dy du
=

= 5u 4 ( 6 x + 1) = 5 ( 3 x 2 + x 1) ( 6 x + 1) .
dx du dx

In composite form,
y = f (u) = u5 and u = g(x) = 3x2 + x 1.

Here, f (u) = 5u4, that is f (g(x)) = f (3x2 + x 1) = 5(3x2 + x 1)4 and


g(x) = 6x +1.
Therefore,
5
d
d
3 x 2 + x 1) =
f ( g ( x ))
(
dx
dx
= f ( g ( x ) ) g' ( x )

= 5 ( 6 x + 1) ( 3 x 2 + x 1) .
4

100 X TOPIC 5 DIFFERENTIATION RULE

ACTIVITY 5.2
Please browse the website to try the question Drill-Chain Rule
http://archives.math.utk.edu/visual.calculus/2/chain_rule.2/index.html

Have you understood? Let us now see the proof of the Chain Rule.

Proof
We prove at the point x = c, that is
h(c) = f (g (c)) g(c)
where h(x) = f (g(x)).
We assume that g(x) g(c) for x which is distinct from c, so
h' (c) = lim

f ( g ( x )) f ( g (c ))

xc
f ( g ( x )) f ( g ( c )) g ( x ) g (c )
= lim

.
x c
xc
g ( x) g (c)
x c

Since g is differentiable at x = c so g is also continuous such that whenever x c


then g(x) g(c). Thus, the above limit becomes
lim

g ( x ) g ( c )

f ( g ( x )) f ( g ( c ))
g ( x ) g (c )

g' (c)

= f ( g ( c ) ) g' ( c ) .
The proof above is incomplete since we have assumed that g(x) g(c). We omit
the possibility where g(x) assumes the value g(c) for x which is distinct from c.

TOPIC 5

DIFFERENTIATION RULE W 101

Table 5.2: Examples of Derivatives of Composite Functions

y' ( x) = f ( g ( x ) ) g'
y = f (g(x))

1.

y=

u = g(x)

1
, x2
x2

y = f (u )

y=

u=x2
u = 3x

2. y = sin 3x

1
u

y = sin u

y= u

u = 2x + x 1

dy du

du dx

y' ( x) =

1
( x 2) 2

y(x) = 3cos3x

y' ( x) =

3 ( x 2 + 1)

3.

y = 2x + x 1

4.

y = cos

3x
2

u=

3x
2

y = cos u

3
3x
y' ( x) = sin
2
2

5.

2
y = tan x
5

u=

2
x
5

y = tan u

2
2
y' ( x) = sec x
5
5

2 x3 + x 1

By considering y = (g(x))n and write u = g(x) we obtain y = un. Therefore, by the


Chain Rule

n
n 1
d
dy dy du
g ( x ) =
=

= n ( g ( x ) ) g' ( x ) .
dx
dx du dx

This formula is known as the Power Rule.


This Power Rule can also be written as

d n
du
u ) = nu n 1 .
(
dx
dx

Observe that the derivative of a power function

d n
x ) = nx n 1
(
dx
is a special case of the Power Rule.

102 X TOPIC 5 DIFFERENTIATION RULE

Example 5.6
f (x) = (3 x3)5
f (x) = (5)(3 x3)6(3x2) = 15x2(3 x3)6.
Based on the given examples, please answer the following questions.

EXERCISE 5.4
Find the derivative of each of the following function:
(a)

f (x) = (3x5 + 7)8

(c)

f ( x) = x +
x

Example 5.7
f (x) = sin3 x

f ( x) = 3sin 2 x

d
( sin x ) = 3sin 2 x cos x.
dx

Example 5.8
g(x) = tan3 x

g' ( x) = 3 tan 2 x

d
( tan x ) = 3 tan 2 x sec x.
dx

(b)

f (x) = (2x4 3)5

TOPIC 5

DIFFERENTIATION RULE W 103

EXERCISE 5.5
Find the derivative of each of the following function:
(a)

f (x) = tan2 x; and

(b)

f (x) = sin2 (3x3 + 6).

Example 5.9

f ( x) = x ( x 2 + 1)

5
5
d
f ( x) = x ( x 2 + 1) + ( x 2 + 1)
dx

= x 5 ( x 2 + 1) 2 x + ( x 2 + 1)
4

= ( x 2 + 1) (10 x 2 + x 2 + 1)
4

= ( x 2 + 1) (11x 2 + 1) .
4

EXERCISE 5.6
Find the derivative of the function f (x) = x3 sin3 x.

104 X TOPIC 5 DIFFERENTIATION RULE

Example 5.10
4
2x +1
f ( x) = 2

3x 5
3

2x +1 d 2x +1
f ( x) = 4 2
2

3 x 5 dx 3 x 5

2
3
2 x + 1 ( 3 x 5 ) 2 ( 2 x + 1) 6 x
= 4 2

2
3x 5
( 3x 2 5)

4 ( 2 x + 1) ( 6 x 2 6 x 10 )
3

( 3x

5)

8 ( 2 x + 1) ( 3 x 2 + 3 x + 5 )
3

( 3x

5)

Next, try to solve the following problem.

EXERCISE 5.7
1.

Find the derivative of the function


f ( x) =

2.

(x

( 3x

2
3

+ 3)

2x)

For each function f and g given below, determine the


derivative of y = ( f D g )( x ) using the chain rule.
(a)

y = f (u) = u4

u = g(x) = x3 + 1

(b)

y = f (u) = (u + 1)2

u = g ( x) =

(c)

y = f (u ) =

u = g(x) = x2 + 2

u
u 1

1
x

TOPIC 5

DIFFERENTIATION RULE W 105

ACTIVITY 5.3
How about if the power of the exponent is negative? Can the Chain
Rule still be used?
To obtain an answer, please refer to the website
http://www.ugrad.math.ubc.ca/coursedoc/math100/notes/derivative/de
riv-quotient.html

5.3

IMPLICIT DIFFERENTIATION

Prior to this, the function in which the derivative is to be determined is already


explicitly given, for example

y = f ( x) =

1
x

or

2
y = f ( x) = x + 1.
3

Both of the functions above may be simply given in the form of equations, such as
xy = 1 and 2x + 3y 3 = 0.
We are lucky that the equations can be easily solved for y, but this does not
always happen. For example, the equation
x + x2y3 + y = 5
cannot be easily solved for y.
We shall study the method of obtaining the derivative of y without first solving for
y. This method is known as the Implicit Differentiation.
We assume that there exists a function y = f (x) which is defined by the respective
equation. For example,
2x + 3y 3 = 0.

106 X TOPIC 5 DIFFERENTIATION RULE

By taking the derivative with respect to x, we obtain


d
d
( 2 x + 3 y 3) = ( 0 )
dx
dx
d
d
d
d
( 2 x ) + ( 3 y ) ( 3) = ( 0 )
dx
dx
dx
dx
2
dy
2 + 3 0 = 0, that is y' = .
3
dx
Notice that this result is the same as the result obtained from

2
y = x + 1.
3
Example 5.11
Given xy = 1.
The derivative is:

d
d
( xy ) = (1) .
dx
dx
However,

d
dy
d
( xy ) = x + y and (1) = 0.
dx
dx
dx
So

xy' + y = 0, that is y' =

y
1
= 2
x
x

This result is the same as the result obtained from

y = f ( x) =

1
x

TOPIC 5

DIFFERENTIATION RULE W 107

Example 5.12
Given x + x2y3 + y = 5.
In order to determine the derivative, we note that from the Power Rule we obtain

d 3
y ) = 3 y 2 y' .
(
dx
So

d
d
x + y + x2 y3 ) = ( 5)
(
dx
dx
d
1 + y' + ( x 2 y 3 ) = 0
dx
d
d
1 + y' + x 2 ( y 3 ) + y 3 ( x 2 ) = 0
dx
dx
2 2
3
1 + y' + 3 x y y' + 2 y x = 0
y' (1 + 3 x 2 y 2 ) = 1 2 y 3 x
Hence,
y' =

1 + 2 y3 x
.
1 + 3x2 y 2

Remark: The Implicit Differentiation may only give y in terms of x and y.


Now we shall consider the use of the Implicit Differentiation in the determination
of the tangent line of the graph of an equation.

Example 5.13
We determine the equation of the tangent line of the graph of the equation

3
x2 + 4y2 = 4 at the point 1,
.
2

108 X TOPIC 5 DIFFERENTIATION RULE

The Implicit Differentiation with respect to x of the above equation results in


2x + 8yy = 0 that is y' =

x
, y 0,
4y

3
1
At the point 1,
.
, the slope of the tangent line is y' =
2
2 3

Hence, the equation of the tangent line is


y

3
1
=
( x + 1)
2
2 3

or
y=

1
2 3

( x + 4).

In actual fact, the graph of the equation is an ellipse and is not a function. Please
see Figure 5.1. However, we can still use the Implicit Differentiation.

Figure 5.1

TOPIC 5

DIFFERENTIATION RULE W 109

Please try the following questions.

EXERCISE 5.8
1.

2.

Find

dy
of each of the following equation.
dx

(a)

4x2y 3y = x3 1;

(b)

x2 + y2 = 4;

(c)

y3 xy2 + cos(xy) = 1;

(d)

x2 + a2y2 = 4a2, a is a constant; and

(e)

xy + sin(xy) = 1.

For each of the following equation, determine the equation of


the tangent line at the specified point.
(a)

x3y + y3x = 30; at (1,3);

(b)


sin (xy) = y; at ,1 ;
2

(c)

y + xy 2 = 5; at (4,1);

(d)

x2 + y3 + 3(x y) = 0; at (2,3); and

(e)

y + cos(xy2) + 3x2 = 4; at (1,0).

110 X TOPIC 5 DIFFERENTIATION RULE

Able to determine the derivative of a given function by using the General


Formula of Differentiation.

Shown the Chain Rule for the determination of the differentiation of a


composite function and the Implicit Differentiation for the differentiation of
an implicit function.

Able to use it to find the equation of the tangent line at a given point.

Topic X Theorems on

Differentiable
Function

LEARNING OUTCOMES
By the end of this topic, you should be able to:
1.

Use the Theorem of the Derivative of an Inverse Function to


determine the value of the derivative o f a given inverse function;
and

2.

Use the Mean Value Theorem and the Rolles Theorem to


determine a particular value in a given interval.

INTRODUCTION

In Topic 5, we have learnt about the uses of the Differentiation Rules and the
Chain Rule to determine various function derivatives. We shall now consider the
determination of the Derivative of the Inverse Function and the use of the Mean
Value Theorem and the Rolles Theorem to determine a particular value in a
given interval.

6.1

THEOREMS OF THE DERIVATIVES OF AN


INVERSE FUNCTION

In Basic Mathematics module, we have learnt about the inverse function. Now,
we shall recapitulate the following:

112 X TOPIC 6 THEOREMS ON DIFFERENTIABLE FUNCTION

1
Suppose it is given the function f (x) = 5x, then its inverse is f 1 ( x ) = x . If the
5
d
1
derivative is determined, we have
f 1 ( x ) ) = .
(
dx
5
Remark: Remember that only a one to one function that has an inverse.
However, we often find it difficult to obtain the inverse of a given function.Thus,
the derivative of an inverse function is hard to find.
Now, we shall learn how to determine the derivative of an inverse function
without first determining the inverse function itself.
Application of the Chain Rule
Do you still remember the Chain Rule in Topic 5? This rule is used to determine
the derivative of a Composite Function. From the Chain Rule

d
f ( g ( x ) ) = f ( g ( x ) ) g' ( x ) .
dx

In the special case for f = g1, we obtain

d
g 1 ( g ( x ) ) = ( g 1 ) ( g ( x ) ) g' ( x ) .
dx

However, g1 is an inverse function of g, that is


g1(g(x)) = x,
such that

d
d
g 1 ( g ( x ) ) = ( x) = 1.
dx
dx

So

( g ) ( g ( x ) ) g' ( x ) = 1
1

TOPIC 6

THEOREMS ON DIFFERENTIABLE FUNCTION W 113

and this results in

( g ) ( g ( x ) ) = g' 1( x )
1

which is equivalent to

( g ) ( z ) = g'
1

( g ( z ))
1

We have seen two ways of determining the derivative of an inverse of a function.


We shall now state the following theorem on the derivative of the inverse of a
function.

Theorem of the Derivative of an Inverse Function (TDIF)


If g is differentiable in an interval I and its inverse function g1 exists, then g1
is differentiable at every x provided that g ( g ( x ) ) 0. We write

( g ) ( x ) = g
-1

( g ( x ))
1

g ( g 1 ( x ) ) 0.

We shall see worked examples of the use of the TDIF.

Example 6.1
The function g(x) = x5 + x is a one to one function and it has an inverse. Obtain
(g1)(2). From the TDIF

( g ) ( 2 ) = g'
1

( g ( 2))
1

Since g(1) = 2, that is g1 (2) = 1, therefore,

( g ) ( 2 ) = g'1(1) .

114 X TOPIC 6 THEOREMS ON DIFFERENTIABLE FUNCTION

However,
g' ( x ) = 3 x 2 + 1 and g' (1) = 4,
therefore,

( g ) ( 2 ) = 14 .
1

Example 6.2
Given that f ( x ) = 5 x 5 + cos x + 3 has an inverse. Determine ( f 1 ) ( 7 ) . From the
TDIF,

( f ) (7) =
1

f ( f 1 ( 7 ) )

However,

( f ) ( 7 ) = 1 since f (1) = 5.1


1

+ cos + 3 = 7.

So

( f ) ( 7 ) = f 1(1) ,
1

while
f ( x ) = 25 x 4 sin x,

therefore,

( f ) ( 7 ) = 25.1
1

1
1
= .
sin 25

Answer the following exercise to test your understanding.

TOPIC 6

THEOREMS ON DIFFERENTIABLE FUNCTION W 115

EXERCISE 6.1

1
1 x
, x 0 show that g ( x) =
, 0 < x 1 is an inverse
x +1
x
1
function of f and vice versa. Use two methods to obtain ( f 1 ) .
2
If f ( x) =

6.2

MEAN VALUE THEOREM AND ROLLES


THEOREM

Geometrically, a non constant continuous function changes sign between 2 inner


points. For a differentiable function, the value of its derivative is zero at the point
where it changes sign. Let such point be c where the tangent line is horizontal.
This idea is said as the Rolles Theorem. While the Mean Value Theorem gives a
general overview of the Rolles Theorem.
The Mean Value Theorem states that if the graph of a function f is continuous in
an interval [a,b] and the tangent line at any point is not vertical, then there exists a
tangent line parallel to the secant line that joins (a, f (a)) and (b, f (b)).
In Figure 6.1a, Figure 6.1b and Figure 6.1c respectively, there exist one, two and
three tangent lines.

Figure 6.1a

Figure 6.1b

116 X TOPIC 6 THEOREMS ON DIFFERENTIABLE FUNCTION

Figure 6.1c

Remember that the graph which has no break represents a continuous function
while the tangent line refers to a differentiable function. We state without proof
the Mean Value Theorem as follows.

The Mean Value Theorem (MVT)


If f is continuous in a closed interval [a,b] and differentiable in an open interval
(a, b) then there exists at least one c in an open interval (a,b) such that

f (c) =

f (b) f (a )
.
ba

Remark: In terms of rate of change, the MVT guarantees the existence of c in an


open interval (a,b) where the instantaneous velocity is equal to the average change
in the interval.
In order to have a clearer understanding, visit the following website:
http://www.math.hme.edu/calculus/tutorials/mean_value/
After you have visited the website, please study the given examples and
test your understanding by trying the exercise that follows.

THEOREMS ON DIFFERENTIABLE FUNCTION W 117

TOPIC 6

Example 6.3
Show that f ( x) = 2 x in the closed interval [1,4] satisfies the conditions of MVT
and obtain the value of c.
Remember that f ( x) = 2 x is continuous in the closed interval

[0, ]

and

differentiable in the closed interval [ 0, ] .


Furthermore,
f ( x) = 2

1
2 x

1
.
x

In the special case f ( x) = 2 x in the closed interval [1,4] satisfies both of the
conditions of MVT. That is, f is continuous in the closed interval [1,4] and
differentiable in open interval (1,4). So according to MVT, there exists c the
open interval (1,4) such that

f (c) =

f (4) f (1)
,
4 1

that is
1
42 2
=
=
c 4 1 3

9
which gives c = .
4

118 X TOPIC 6 THEOREMS ON DIFFERENTIABLE FUNCTION

Study Figure 6.2 carefully in order to have a better picture.

Figure 6.2

EXERCISE 6.2
The following functions satisfy the conditions of the Mean Value
Theorem in the specified intervals. Obtain the appropriate value of c.
(a)

f (x) = (x 1)3 in [1,1]

(b)

g ( x) =

x2
in [0,1]
x +1

Example 6.4
Show that f (x) = x2 5x + 6 satisfies the MVT in the closed interval [1,4] and
obtain the appropriate value of c.
Remember that a polynomial function is continuous and differentiable in any
interval. So the conditions of the MVT are satisfied by f (x) = x2 5x + 6.

TOPIC 6

THEOREMS ON DIFFERENTIABLE FUNCTION W 119

According to the MVT, there exists c in the open interval (1,4) such that

f (c) =

f (4) f (1)
.
4 1

Since f (4) = 2 = f (1) and f (x) = 2x 5,

5
Therefore, 2c 5 = 0, that is c = .
2
Consider Figure 6.3

Figure 6.3

In example 6.4, the function f is continuous in the closed interval [1, 4] and
differentiable in the open interval (1,4). In addition, the two end values of f are
the same, that is f (4) = f (1). In such a case, the MVT is known as the Rolles
Theorem.

Rolles Theorem (RT)


If f is continuous in a closed interval [a,b] and differentiable in (a,b) and in
addition f (b) = f ( a), then there exists c in an open interval (a,b) such that
f (c) = 0.

120 X TOPIC 6 THEOREMS ON DIFFERENTIABLE FUNCTION

So RT is a special case of the MVT since if f (b) = f (a) then the MVT guarantees
the existence of c in (a,b) such that

f (c) =

f (b) f (a )
=0
ba

Example 6.5
By considering f (x) = (x 1) sin x in the closed interval [0,1], show that the
equation tan x + x =1 has root in the open interval (0,1).
Solution
The function f is continuous in the closed interval [0,1] and differentiable in the
open interval (0,1) since f is the multiplication of two any where differentiable
functions. In addition, f (0) = 0 = f (1). So according to the Rolles Theorem, there
exists c in the open interval (0, 1) such that f (c) = 0. That is,
(c 1) cos c + sin c = 0.
In the interval (c 1), cos c 0, so
tan c + c 1 = 0.
This shows that there exists c in the open interval (0,1) that forms the solution of
tan x + x =1.

Example 6.6
Find the value of c satisfying the conditions of the Rolles Theorem for
x3 x2 x + 1 on the interval [0,2].
Solution:

1
Let f (x) = 3x2 2x 1 f (c) = 3c2 2c 1 = 0 (3c + 1)(c 1) = 0 c = 1, .
3
Since the [0,2], the appropriate value of c = 1.

TOPIC 6

THEOREMS ON DIFFERENTIABLE FUNCTION W 121

EXERCISE 6.3
The functions below satisfy the conditions of the Rolles Theorem in
the specified intervals. Obtain the appropriate value of c.
(a)

f (x) = x3 x + 2 in [1,1]

(b)

g(x) = sin x + cos x in [0, 2]

To end our discussion, we can interpret the Mean Value Theorem and the Rolles
Theorem physically as follows.

The Mean Value Theorem


If f (t) denotes the distance an object traversed in time t, then

f (b) f (a )
ba
represents the average velocity in the time interval [a,b].
Thus the existence of c (a,b) such that

f (c) =

f (b) f (a )
ba

exerts that the object is guaranteed to achieve its average velocity.

Rolles Theorem
Since f (a) = f (b) denotes that the object is at the same location at two distinct
instants, therefore there exists c (a, b) with f (c) = 0. This exerts that the object
is guaranteed to a stop at some instant in the time interval (a,b).
You may need to study the graphical examples and the Mean value Theorem
at the following website:
http://www.fsas.upm.edu.my/~suriah/nota/k16.pdf

122 X TOPIC 6 THEOREMS ON DIFFERENTIABLE FUNCTION

In Topic 6, we have learnt the method of determining the differentiation of an


inverse function using the Theorem on the Derivative of an Inverse Function.
The Mean Value Theorem and the Rolles Theorem are used to obtain a value
of c in a given interval.

Topic X Various Rates

of Change

LEARNING OUTCOMES
By the end of this topic, you should be able to:
1.

Determine the relation from a given problem; and

2.

Define the derivative and solve the related problem.

INTRODUCTION

In the previous topic, we have learnt the methods of determining the derivative of
a given function. Now, we shall consider how to use derivative to determine the
rate of change of a moving object.

7.1

RATE OF CHANGE

We know that the derivative of a function f with respect to x which is usually


df
written as
or f (x) gives the rate of change of the quantity f with respect to x.
dx
df
or f (t) is said as the rate of change of f with
If f depends on the time t, then
dt
respect to time.
We refer to Table 7.1 to see several functions and their respective derivatives.

124 X TOPIC 7 VARIOUS RATES OF CHANGE

Table 7.1
Function f (t)
Distance
Velocity
Population Density
Radioactive Amount
Amount of Chemical Material

Function f (t)
Speed
Acceleration
Growth Rate
Rate of Decay
Rate of Reaction

Next, we shall study the following examples.


Example 7.1
A boy, whose height is 1 m., walks past a lamp post with a speed of 1 m/s. If the
height of the lamp post is 6 m, what is the speed of the boys shadow?
Solution
We assume that the surrounding is flat and lighted by the lamp from the top of the
lamp post. Suppose that x(t) is the distance walked by the boy and y(t) is the
distance covered by the shadow. The position of the boy and the shadow is
illustrated in Figure 7.1.

Figure 7.1

Observe that

TOPIC 7

tan =

y
6

VARIOUS RATES OF CHANGE W 125

and tan = y x

Therefore,

yx=

y
6
, that is y = x
6
5

The derivative with respect to t gives the speed of the shadow as

dy 6 dx 6
=
= m/s
dt 5 dt 5
Example 7.2
A plane is flying at a height of 12 km. and past a radar station. When the plane is
20 km. away from the station, it is scanned that the distance increases at the rate
of 400 km/hr. Determine the speed of the plane.
Solution
We make a trace of the flight of the plane as in Figure 7.2.
Suppose that is the horizontal distance of the plane away from the station.

Figure 7.2

126 X TOPIC 7 VARIOUS RATES OF CHANGE

Suppose that x is the horizontal distance of the plane away from the station.
Observe that
122 + x 2 = z 2
2 xx = 2 zz
z
x = z ,
x
but if z = 20, x = 202 122 = 16.
Therefore,

x' =

20
(400) = 500 km / hr.
16

Thus the speed of the plane at that instant is 500 km/hr.


Examples 7.1 and 7.2 illustrate that derivative as the rate of change of the function
of distance with respect to time.
ACTIVITY 7.1
Suppose it is given that the length of the sides of a square is s feet and
ds
is the rate of change of s in inch/minute, how are we going to
dt
determine the rate of change of the area?

Now, do the following exercise to test your understanding.

TOPIC 7

VARIOUS RATES OF CHANGE W 127

EXERCISE 7.1
A car A is driven to the west with a speed of 70 km/hr. while another
car B is driven to the north with a speed of 90 km/hr. Both cars are
driven towards a junction of the two roads. Determine the rate of
change of the distance between the two cars when the car A and the
car B are respectively at 0.4 km. and 0.3 km. away from the junction.

Example 7.3
The population number of a certain species of rabbit changes logistically, that is if
y = ay(ym y)
where a is a positive constant and ym is the maximum number of rabbits that can
be accommodated in a specified environment.
Explain the equation above and show that
y (t ) =

y ( 0 ) ym

y ( 0 ) + ( ym y ( 0 ) ) e aymt

and sketch its graph.

Solution
Note that the equation y = ay (ym y) shows that
(i)

y approaches to 0 as y approaches to ym, that is the population growth is


small if the population number is sufficiently large.

(ii)

y < 0 if y > ym, that is there occurs a reduction in population.

(iii) when y is small (compared to ym), that is (ym y) ym, the equation becomes
y aymy.

128 X TOPIC 7 VARIOUS RATES OF CHANGE

The equation is satisfied by


y (t ) = ke aymt .
From

y ( 0 ) ym

y (t ) =

y ( 0 ) + ( ym y ( 0 ) ) e aymt

y ( 0 ) ym aym ( ym y ( 0 ) ) e aymt
dy
=
;
2
dt
y ( 0 ) + ym y ( 0 ) e aymt
ay ( 0 ) ym2 ( ym y ( 0 ) ) e aymt
.
y' =
2
y ( 0 ) + ( ym y ( 0 ) ) e aymt

While

ay ( ym y ) =
=

ay ( 0 ) ym

y ( 0 ) + ( ym y ( 0 ) ) e

aym t

ay ( 0 ) ym2 ( ym y ( 0 ) ) e aymt

y ( 0 ) + ( ym y ( 0 ) ) e aymt

ym y ( 0 ) + ( ym y ( 0 ) ) e aymt y ( 0 ) ym

y ( 0 ) + ( ym y ( 0 ) ) e aymt

Thus, the equation y' = ay ( ym y ) is satisfied.


The graph of y' = ay ( ym y ) is given in Figure 7.3.

TOPIC 7

VARIOUS RATES OF CHANGE W 129

Figure 7.3

Remark: In actual fact, the function y which gives the population number is not a
differentiable function. Furthermore, it is not even continuous and the graph is just
some finite numbers of points. However, for large number of population, you can
assume y to be continuous and if there does not occur any abrupt change say a
mass migration and natural disaster, y can be assumed as differentiable.
EXERCISE 7.2
1.

Two resistors with resistances R1 and R2 are connected in


parallel such that the total resistance R is given by

1 1
1
= +
R R1 R2
If R1 and R2 increase at the rate of 0.2 /s and 0.3 /s
respectively, determine the rate of change of R when R1 = 100
and R2 = 100 .
2.

The height of a triangle increases at the rate of 1 cm/s, while its


area increases at the rate of 13 cm2/s. Obtain the rate of change
of the base when it is given the area of the triangle is 180 cm2
and the height is 18 cm.

130 X TOPIC 7 VARIOUS RATES OF CHANGE

In Topic 7 we have learnt how to solve problems involving rate of change by


taking the following steps: sketch the graph; determine the relation based on
the given properties; determine the problem to be solved; and use the
Differentiation Rule to obtain the rate of change required.

Visit the websitehttp://www.fsas.upm.edu.my/~suriah/bab4.html to answer the


activities concerning the rate of change.

Topic X Application in

Economics

LEARNING OUTCOMES
By the end of this topic, you should be able to:
1.

Determine the unit cost of a product; and

2.

Determine the marginal cost given the production cost.

INTRODUCTION

In economics, there are many rates of change that are not measured with respect
to time. For example, the economist usually refer the marginal profit and the
marginal cost respectively as the rate of change of the profit and cost with respect
to the quantity of products produced or sold. In this chapter we shall see the
contribution of calculus in the Economics Theory, that is in the topics of marginal
profit and the production cost.

8.1

MARGINAL PROFIT AND COST


FUNCTIONS

Suppose that a manufacturing firm produces x units of a certain product and


market them out at the price of p(x) per unit.
Remark: Notice that the price of a unit of good is dependent on the quantity of x.
This is because, as might be expected, that the firm may raised the price whenever
there is an increased in the demand. In the case when there is an excess of
production, the firm may have to lower the price.

132 X TOPIC 8 APPLICATION IN ECONOMICS

The profit of the firm, K, is given by the difference between the selling price and
the production cost, C, that is:
K(x) = Selling Price Cost = xp(x) C(x)

where we use the following notations:


x Quantity of product produced;
C Production cost;
c Production cost per unit;
p Price per unit; and
K Total profit.
How about the functions C and p?
The cost function C (say in RM) may be of the linear form:
C(x) = 20000 + 10x

Here, RM20 000 represents a fixed cost while RM10x is a variable cost based on
RM10 for every unit of product produced. However, it is more obvious that the
cost does not increase linearly.
May be

C ( x) = 20 000 + 10 x + 100 x
is more meaningful because of the factor

Now consider the function p, that is the price per unit of product (say in RM) may
be of the form

p ( x) = 15

x
, x 20 000.
10 000

TOPIC 8

APPLICATION IN ECONOMICS W 133

Here, the price per unit of product is RM15 if production does not exceed 20 000
units. For every additional 10 000 units of product, the price per unit is reduced by
RM1. In this case the profit function is:

K ( x) = xp ( x) C ( x)

x
= x 15
20 000 + 10 x + 100 x
10 000

x2
= 5 x 20 000
100 x .
10 000

We shall now consider some examples.


Example 8.1
Suppose the production cost of x units of product is given by

C ( x) = 20 000 + 10 x + 100 x
and the price per unit of product (also said as the demand function) is

p ( x) = 15

x
.
10 000

Determine the total profit and the marginal profit for the production of 22 000
units, 23 300 units and 25 000 units of the specified product.
Solution
x2
K ( x) = 5 x 20 000
100 x .
10 000
The marginal profit

K' ( x) = 5

x
50

.
5000
x

134 X TOPIC 8 APPLICATION IN ECONOMICS

The total profit and the marginal profit are given in Table 8.1
Table 8.1
22 000

23 300

Profit

26 767.60

26 946.67

26 688.61

Marginal Profit

0.263

0.012

0.316

25 000

Since
K ( x + x ) K ( x )
x 0
x

K' ( x) = lim

therefore, for x = 22000 and x = 1, we obtain


K(22000) K (22000 + 1) K (22000),
that is K (22000) gives the profit approximation in the production increment from
22000 units to 22001 units.
ACTIVITY 8.1
From the given example above, how will it affect the firm if
production exceeds 23300 units?

Example 8.2
A firm approximates the cost (in thousands of RM) per unit in the production of x
units of a specified product as given by

c( x) = 0.04 + 0.0002 x +

3200
.
x

Determine the marginal cost per unit of production of 1000 units, 4000 units and
5000 units of product and give your comments.

TOPIC 8

APPLICATION IN ECONOMICS W 135

Solution:
The marginal cost per unit
3200
x2
c' (1000) = RM(0.003 103 )
c' ( x) = 0.0002

c' (4000) = RM0


c' (5000) = RM0.072
This means that an increase of 1 unit in the production from 1000 units to 1001
units reduces the cost per unit an amount of RM3. However, an increase of 1 unit
of production from 4000 units to 4001 units does not affect the cost per unit. Next,
an increase of 1 unit from 5000 units to 5001 units results to an increase in the
cost per unit of RM0.072. The calculation above shows that the production of
4000 units results in the cost per unit to remain in the lowest level.

EXERCISE 8.1
1.

Suppose the cost function (RM) of producing x units of a certain


product is
1
3

K ( x) = 8300 + 3.25 + 4 x .

Determine the average cost per unit and the marginal cost given
x = 1000.
2.

Suppose that the price per unit and the cost of production of x
units are respectively given by
p(x) = 5.00 0.002x,
C(x) = 3.00 + 1.10x.
Obtain the marginal income, marginal cost and the marginal
profit if the production x = 800, 975 and 1100.

136 X TOPIC 8 APPLICATION IN ECONOMICS

We have learnt about the profit function as

K(x) = Selling Price Cost Price = xp(x) C(x).

The derivative of the profit function is said as the marginal profit.

Topic X Approximation

LEARNING OUTCOMES
By the end of this topic, you should be able to:
1.

Use an appropriate approximation method to find the estimate value


of a given mathematical problem.

INTRODUCTION

In the previous topic we have learnt about rate of change and derivatives. In this
topic, we shall discuss methods of finding the approximate solution of more
difficult problems using derivatives. There are 4 methods of approximation that
we shall discuss here; they are the methods of Differentiation,
Linear Approximation, Taylor and Quadratic Approximation and Newton
Approximation.

9.1

DIFFERENTIAL

Estimation of a Change
As we have learnt, the derivative of a function y = f (x) is given by
f ( x) =

f ( x + x ) f ( x )
dy
y
.
= lim
= lim
dx x 0 x x 0
x

dy
as the derivative of y with respect to
dx
x. We shall now give a different meaning to the symbols dy and dx.
Note that so far, we interpret the notation

138 X TOPIC 9 APPROXIMATION

Note that

y
f ( x)
x

as

x 0.

By writing differently,

y f ( x)x

as x 0.

In actual fact, the quantity f (x) x is symbolized by dy that is known as the


differential.
We obtain the definition of the approximate differential as:
Suppose that y = f (x) is a differentiable function. The differential of y which is
denoted with dy is given by f (x) dx where dx = x 0 is called as the
differential of x.

SELF-CHECK 9.1
Try to obtain some examples of situations where differential
approximations are used.

Errors of Differential Approximation


Consider Figure 9.1 which illustrates the difference between dy and y. Note also
that in Figure 9.1a, y is larger than dy, while in Figure 9.1b, y is smaller than
dy.

TOPIC 9

(a)

APPROXIMATION W 139

(b)

Figure 9.1 (a) & (b)

Notice that dy is dependent on two variables, x and dx. For example, the
differential of the function y = x2 is dy = 2xdx. Therefore, if
x = 1 and dx = x = 0.5 then dy = 2(1)(0.5) = 1,
x = 1 and dx = x = 0.1 then dy = 2(1)(0.1) = 0.4,
x = 1 and dx = x = 0.3 then dy = 2(1)(0.3) = 0.6.
Now consider the following examples.
Example 9.1
Use the differential to estimate the increments in the volume and the surface area
of a ball if the radius increases from 3 cm to 3.01 cm.
Solution:

4
Refer to Figure 9.2. The volume of the ball is I = j 3 and dI = 4 j2 dj.
3
If j increases from 3 cm to 3.01 cm, dj = j = 0.01.
Therefore, the increment in the volume
dl = 4 (32 )(0.01) 1.131 cm3
The surface area L = 4 j2 and dL = 8 j dj.

If j increases from 3 cm to 3.01 cm, dj = j = 0.01.

140 X TOPIC 9 APPROXIMATION

Therefore, the increment in the surface area is


dL = 8 j dj = 8 (3)(0.01) 0.754.

Figure 9.2

Example 9.2
The length of the sides of a cube is measured to be 22.1 cm with an error of
0.05 cm. Determine the volume of the cube and estimate its error.
Solution:
Refer to Figure 9.3. The volume of the cube I = x3 and dI = 3x2dx.
If x = 22.1 and dx = x = 0.05, then
I = 22.13 10793
and

dI = 3(22.1)2(0.05) 73.

Therefore, the volume of the cube should be written as 10793 73cm3.

Figure 9.3

TOPIC 9

APPROXIMATION W 141

ACTIVITY 9.1
Refer the website to answer the activities given. http://www.
ohaganbooks.com/StudentSite/calctopic1/linearapptox.html

Try out the following questions.


EXERCISE 9.1
1.

Use the differential to estimate the volume of rubber needed to


make a ball of radius 8 cm. with thickness of 0.1 cm.

2.

The angle between two adjacent sides of equal length of an


isosceles triangle is measured to be 0.50 0.005 radians. The
length of the two sides of equal length is found to be exactly
120 cm. Calculate
(a)

the length of the third side and its error.

(b)

the area of the given triangle and its error.

142 X TOPIC 9 APPROXIMATION

9.2

LINEAR APPROXIMATION

Refer back to Figure 9.1 that illustrates


f ( x + x ) f ( x) + dy = f ( x) + f ( x) x

though it is true that f ( x + x ) = f ( x) + y. This shows that we can obtain the


estimate of f ( x + x ) using the differential dy = f (x)dx (dx = x).
Example 9.3
Suppose that f (x) = x3 + x. We can estimate the value of f (x + x) using

f ( x + x ) f ( x) + ( 3 x 2 + 1) x.
Thus, in the special case we write 2.01 as 2 + 0.01, we obtain

f (2.01) = f (2 + 0.01) f (2) + 3 ( 2 ) + 1 ( 0.01)


2

= ( 23 + 2 ) + 0.13 = 10.13.

This value does not differ much from the true value of f at 2.01, that is
f (2.01) = 2.013 + 2 = 10.1201.
However, a distinguishable error might occur if x is large enough. For example,
suppose that x + x = 2.3 = 2 + 0.3, we have

f ( 2.3) = f (2 + 0.3) f (2) + 3 ( 2 ) + 1 ( 0.3) = 13.9.

While the true value of f at 2.3 is


f (2.3) = 2.33 + 2.3 = 14.467.

TOPIC 9

Example 9.4
Suppose that we want to obtain the value of
the mathematical table.
Solution:

APPROXIMATION W 143

80 without using the calculator or

We can estimate the value of g(x + x) using


g(x + x) g(x) + g(x) x.
In this case,
1

g ( x ) = 4 x = x 4 and g' ( x ) =

1
4x

Since,

3
4

, x > 0.

81 = 3, we write 80 = 81 + (1) such that


g (80) = 4 80 g (81) + g' (81)(1)
1

= 4 81 +
= 3

4(81)
1

4(3)
1
= 3
108
107
=2
.
108

3
4

(1)

3
4

If you use a calculator, you will obtain 2.990741 (7 digits) as the approximation.
The true value of 4 80 approximated to 7 digits is 2.990698.
The approximation method illustrated in the two examples above is known as the
Method of Linear Approximation. The rational behind this name is illustrated in
Figure 9.4.

144 X TOPIC 9 APPROXIMATION

tangen line

Figure 9.4

The equation of the tangent line that passes through the point (a, f (a)) is
y f (a) = f (a)(x a)
or
y = f (a) + f (a)(x a).
Note that the point (x, f (x)) is on the graph of f while the point (x, f (a) + f (a)
(x a)) is on the tangent line.
We obtain the linear approximation as:
In the situation where x approaches to a, the value of f (x) can be obtained as
f (a) + f (a)(x a)
which in actual fact is the value of y on the straight line (linear).

The Linear Approximation Method is also called as the method of differential.

TOPIC 9

APPROXIMATION W 145

EXERCISE 9.2
Use the Linear Approximation Method to determine:

16.6,

(a)
(b)

(c)

65 3 ,

(d)

cos 29,

(e)

sin 32.

8.8,
2

9.3

TAYLOR AND QUADRATIC


APPROXIMATION

In Section 9.2 we have learnt that the linear approximation of the function f
about x = a is given by
y = f (a) + f (a)(x a).
Here y is dependent on x and now we write
P1(x) = f (a) + f (a)(x a).
Note that P1(x) is a polynomial of degree 1 and is the tangent line of f at x = a.
The polynomial P1(x) is a polynomial that approximates f (x) as x approaches to
a.
Can we obtain another polynomial, say P2(x) which gives a better approximation
to f (x) as compared to P1(x)?

Polynomial of Degree 2
We search a polynomial of degree 2 which has the following properties at x = a:
(i)

its tangent line is the same as the tangent line of f.

(ii)

its second derivative is the same as that of f.

146 X TOPIC 9 APPROXIMATION

The polynomial we are searching is

P 2 ( x) = f (a ) + f (a )( x a) +

f (a )
( x a)2
2

This is because
(a)

P2 ( a ) = f ( a )
The value of P2 (a) and the value of f (a) are the same.

(b)

P2(x) = f (a) + f (a)(x a) such that P2(a) = f (a).


The slope of the tangent lines of P2 and f are the same at x = a.

(c)

P2(x) = f (x), in particular P2(a) = f (a).


The second derivatives of P2 and f are the same at x = a.

In Section 9.2 we have obtained that the linear approximation of f (x) = x3 + x is


given by

P1(x) = f (a) + (3x + 1)(x a).


In Particular, at a = 2

P1(x) = f (2) + (3x2 + 1)(x 2).

Example 9.5
In Section 9.2, we have shown that
P1(2.01) = 10.13 and P1(2.3) = 13.9
while the true value is

f (2.01) = 10.120601 and f (2.3) = 14.467.


Since f (x) = 6x, therefore,

P2(x) = f (a) + (3x2 + 1)(x 2) + 3x(x 2)2.

TOPIC 9

APPROXIMATION W 147

Thus

P2(2.01) 10.1306 and P2 (2.3) = 14.44.


Note that

P2(2.01) f (2.01) = 10.1306 10.120601 = 0.009999.


and

P1(2.01) f (2.01) = 10.13 10.120601 = 0.009399.


Next,

P2 (2.3) f (2.3) = 14.44 14.467 = 0.027


and
P1 (2.3) f (2.3) = 13.9 14.467 = 0.567.

Thus
(a)

P1(2.01) is closer to the true value f (2.01) as compared to that of P2(2.01).


However, both are good approximation of f (2.01).

(b)

P2(2.3) is closer to the true value f (2.3) as compared to that of P1(2.3).

SELF-CHECK 9.2
Can you make any conclusion about the better approximation if x is
sufficiently large.

The approximation of f (x) using P2(x) is known as the Quadratic Approximation


since the polynomial P2(x) is of degree 2.

148 X TOPIC 9 APPROXIMATION

Polynomial Approximation of Degree 3


We have learnt that about the point x = a, the value of f (x) can be estimated by the
polynomial of degree 2, that is
P2 ( x) = f (a) + f (a )( x a ) +

f (a )
( x a)2 .
2

Based on the same argument, we can show that f (x) can be estimated by the
polynomial of degree 3, that is
P3 ( x) = f (a ) + f' (a )( x a) +

f'' (a )
f (a )
( x a) 2 +
( x a )3 .
2!
3!

Consequently, f (x) can be estimated by a polynomial of degree n given by:

Taylors Polynomial
Pn ( x) = f (a) + f (a )( x a ) +

f (a )
f ( n ) (a )
( x a ) 2 + ... +
( x a)n
2!
n!

Here, f (n)(a) is the nth derivative of f taken at x = a.

The polynomial Pn(x) is known as the Taylors polynomial of degree n at x = a.


Consequently, the estimation of f (x) by Pn(x) is called the Taylors
Approximation.

Example 9.6
Estimate the value of

0.8 using P1(x), P2(x) and P3(x). Note that

f ( x) = x
f' ( x) =

2 x
1
f'' ( x) = 3
4x 2
3
f''' ( x) = 5
8x 2

f (1) = 1

f' (1) =

f'' (1) =

f''' (1) =

1
2

3
8

1
4

TOPIC 9

APPROXIMATION W 149

Therefore,

P1 ( x) = f (1) + f (1)( x 1)
1
P1 (0.8) = 1 + (0.8 1) = 0.9
2
P2 ( x) = f (1) + f (1)( x 1) +

f (1)
f (1)
2
2
( x 1) = P1 ( x) +
( x 1)
2
2

1 1
2
P2 (0.8) = P1 (0.8) + ( 0.2 ) = 0.895
2 4
f (1)
3
P3 ( x) = P2 ( x) +
( x 1)
3!
13
3
P3 (0.8) = P2 (0.8) + ( 0.2 ) = 0.8945.
68
Note that the values of P1(0.8), P2(0.8) and P3(0.8) are getting closer to the true
value of f (0.8) = 0.8 , that is 0.89443 (5 decimal digits accuracy).

EXERCISE 9.3
1.

2.

Write the Taylors polynomial of degree n about a for the


following functions.
(a)

f ( x) = 3 x

(b)

f ( x) =

a =1

n=5

a=0

n=2

(c)

1
1 + x2
f ( x) = cosh x

a=0

n=6

(d)

f ( x) = cos x

a=0

n=6

Use the results in (1) to estimate


(a)
(b)
(c)
(d)

1.3,

1
,
1 + 0.22
cosh (0.3),
cos 0.2.

150 X TOPIC 9 APPROXIMATION

9.4

NEWTON METHOD

Newton Method is a technique to estimate the zero (real) of a function. This


method uses the tangent line (linear approximation) as the approximation of f.
See Figure 9.5.

tangent line y-f (x1) = f ' (x1)(x x 1)

x1

x3

x2

tangent line y f (x 2) = f ' (x2)(x x2)

Figure 9.5

Suppose that f is continuous in a closed interval [a,b]. If f (a) and f (b) are of
opposite signs, then the Mid Value Theorem guarantees the existence of the zero
of f in the open interval (a,b). Let us assume that the zero be at x = x1.
If f is also differentiable in (a,b), then f (x) exists and the equation of the tangent
line of f that passes through the point (x1, f (x1)) is
or

y f (x1) = f (x1)(x x1)


y f (x1) + f (x1)(x x1).

This line intersects with the x-axis, that is the line y = 0, if

x = x1

f ( x1 )
.
f ( x1 )

Thus, from the first estimate x1 we obtain the second estimate of the zero of f.

TOPIC 9

APPROXIMATION W 151

Similarly, we can obtain the third estimate x3 by considering the tangent line of f
at the point (x2, f (x2)),
y f (x2) = f (x2)(x x2).
This line cuts the x-axis if

x = x3 = x2

f ( x2 )
.
f ( x2 )

ACTIVITY 9.2
Refer the website http://www.sosmath.com/calculus/diff/der07/
der07.html and obtain further explanation. Try the given activities.

Example 9.7
Use the above method to estimate the zero of f (x) = x2 2.
Solution:
The polynomial f is continuous in any interval and is also differentiable.
Since f (0) = 2 < 0 and f (2) = 4 > 0, the Mid Value Theorem guarantees the
existence of the zero of f in the open interval (0,2). We assume the first estimation
x1 = 1. Hence, the second estimation is
12 2 )
(
f ( x1 )
= 1
= 1.5.
x2 = x1
f ( x1 )
2(1)
Consequently, the third estimation is
1.52 2 )
(
f ( x2 )
x3 = x2
= 1.5
= 1.41667.
f ( x2 )
2(1.5)

152 X TOPIC 9 APPROXIMATION

Further calculation results in the fourth estimation as


1.41667 2 2 )
(
f ( x3 )
x4 = x3
= 1.41667
= 1.4142
f ( x3 )
2(1.41667)
points)

(4

demical

The method of obtaining the estimation of the zero of a function as illustrated in


the above example is known as the Newton Method. Note that the method
involves the division by f (x1). Hence, the condition that f (x1) is not zero is
important otherwise the method fails. However, this can be overcome by taking a
different estimate of x1.
Furthermore, we have assumed that f is differentiable at every point in the
interval under consideration, otherwise, the method may produce a divergent
result. For example, consider
1

f ( x) = x 3 with x1 = 0.1.
In this case,
1 2
f ( x) = x 3
3
and
1

( 0.1) 3 = 0.2.
f ( x1 )
= 0.1
x2 = x1
2
1

f ( x1 )
0.1
( )3
3
Consequently, the third estimation is
1

( 0.2 ) 3
f ( x2 )
x3 = x2
= 0.2
= 0.4.
2
1

f ( x2 )
3
0.2

( )
3
1
3

Notice that the estimation is getting further away from the zero of f ( x) = x , that
is x = 0!

TOPIC 9

APPROXIMATION W 153

This is illustrated in Figure 9.6.

Figure 9.6

EXERCISE 9.4
1.

2.

Use the Newton Method to obtain the second estimation of the


zero of the function with the given initial estimation.
(a)

f ( x) = x 5

x1 = 2

(b)

f ( x) = cos x

x1 = 1.5

(c)

f ( x) = 3 x 1 x

x1 = 7

Use the Newton Method to obtain the estimation of the


x-abscissa of the point of intersection between f (x) = 3 x and
1
g ( x) = 2
. Continue the process until the difference between
x +1
two consecutive estimations is less than 0.001.

154 X TOPIC 9 APPROXIMATION

Method of Differential to determine dy as f (x)dx where dx = x (0) is called


the differential of x.

Linear Approximation
f (a) + f (a)(x a).

Quadratic Approximation
f (a ) + f (a)( x a ) +

Taylors Approximation
f (a ) + f (a)( x a) +

f (a)
( x a)2 .
2

f (a )
f n (a)
( x a) 2 + ... +
( x a)n .
2!
n!

Newton Method to determine the approximate root of an equation using the


tangent line.

Topic X Extremum

10

Problem and
the Tracing of
a Graph

LEARNING OUTCOMES
By the end of this topic, you should be able to:

1.

Define local extremum and critical point of a function;

2.

Describe the tracing of a graph; and

3.

Describe the method of solving applied extremum problems.

INTRODUCTION

In this topic, you will be exposed with the concept of local extremum and the
tracing of a graph.

10.1

EXTREMUM AND LOCAL EXTREMUM

The largest value of a function in an interval is its maximum, while the smallest
value of a function is its minimum. Both of these values are known as the absolute
maximum and minimum.

156 X TOPIC 10 EXTREMUM PROBLEM AND THE TRACING OF A GRAPH

Now, we have the following definition:


Definition of Maximum
Suppose f is defined in an interval I that contains c, then f (c) is the maximum
of f in I provided f (c) f (x) for every x in I.

ACTIVITY 10.1
Using your own words, explain and define the meaning of minimum.

Sometimes, a function may only have a maximum (without a minimum) or


otherwise. There are also functions that do not have both maximum and
minimum. Consider for example Figure 10.1.
(a)

(b)

TOPIC 10

(c)

EXTREMUM PROBLEM AND THE TRACING OF A GRAPH W 157

(d)

Figure 10.1 (a), (b), (c) & (d)

In Figure 10.1a, the function h does not attain its maximum and minimum in the
open interval (1,2). In Figure 10.1b, the continuous function f does not attain its
maximum but has a minimum in the open interval. In Figure 10.1c, the
discontinuous function has no minimum but has a maximum in the closed
interval. Finally, in Figure 10.1d, the continuous function attains both its
maximum and minimum in the closed interval.
By referring the maximum and minimum as extremum, the situation above can be
summarised in the following theorem:
The Extremum Value Theorem
If f is continuous in a closed interval, then f attains both its maximum and
minimum in he specified interval.

Now, we shall learn about local (relative) maximum and local (relative)
minimum. Consider Figure 10.2 that illustrates the graph of a function f in a
closed interval [x1, x8].

158 X TOPIC 10 EXTREMUM PROBLEM AND THE TRACING OF A GRAPH

Figure 10.2

Consider in the neighborhood of x = x6. We can take an open interval small


enough to contain x6 such that the value of the function at x6, f (x6), is the largest
value of f in the interval. In this case, f (x6) is said to be the local maximum of f
.
Definition of Local Maximum
If there is an open interval I containing c such that f (c) is the maximum of f in
the interval, that is:
f (c) f (x)

x I,

then f (c) is the local maximum of f.

Note that f (x3) is also a local maximum of f since there is an open interval J
containing x3 such that f (x3) f (x), x J. Similarly, f (x2) is also a local
maximum.
Next, we define a local minimum as follows:
Definition of Local Minimum
If there is an open interval I containing c such that:
f (c) f (x)

x I,

then f (c) is the local minimum of f.

In Figure 10.2, f (x4), f (x7) and f (x1) are also local minimum of f.

TOPIC 10

EXTREMUM PROBLEM AND THE TRACING OF A GRAPH W 159

ACTIVITY 10.2
Show or prove that f (x4), f (x7) and f (x1) are local minimum of f.

Next, f (x5) is not a local minimum since for any open interval containing x5, we
cannot find that
f (x5) f (x) for every x in the interval.
In actual fact,
f (x5) < f (x) only for x to the right of x5.
but
f (x5) > f (x) for x to the left of x5.
Remark: Finally, remember that local maximum and local minimum cannot
occur at the ends of an interval. This is because every open interval that contains
those end points; f is only defined at some parts of the interval. It may be defined
only on the left part or the right part.
Locations of Local Maximum and Local Minimum
Let us consider Figure 10.2. It shows that:
f (x2) is a relative maximum, f (x2) does not exist
f (x3) is a relative maximum, f (x3) = 0
f (x4) is a relative minimum, f (x4) does not exist
f (x6) is a relative maximum, f (x6) = 0
f (x7) is a relative minimum, f (x7) = 0.

160 X TOPIC 10 EXTREMUM PROBLEM AND THE TRACING OF A GRAPH

SELF-CHECK 10.1
What can you conclude from your observation of Figure 10.2?

Now, we obtain the definition of Critical Point, that is:


Definition of Critical Point
Suppose that f is defined at c. If f (c) = 0 or f (c) does not exist, then (c, f (c))
is called the critical point.

In other words, a critical point is a location where f takes the value 0 or is not
defined. We shall consider the following examples to determine the critical point
of a function.
Example 10.1
f (x) = 2x3 + 3x2
Solution:
Here f (x) = 6x2 + 6x = 6x (x +1)
and f (x) = 0 if x = 0 or x = 1.
Therefore, (0,0) and (1,1) are the critical points of f.
Example 10.2
2

g ( x) = x 3

Solution:
Here g' ( x) =

2
3x

1
3

, x 0, that is g does not exist at x = 0

Therefore, (0,0) is the only critical point of g.

EXTREMUM PROBLEM AND THE TRACING OF A GRAPH W 161

TOPIC 10

Example 10.3
2

h( x ) = 2 x 3 x 3 .

Solution:
Here h' ( x) = 2

2
x

1
3

, x 0, that is h(0) does not exist and h(1) = 0.

Therefore, the critical points of h are (0,0) and (1,1).

EXERCISE 10.1
Determine the critical point of each of the function in the specified
interval:
(a)

f ( x) = 2 x 3 15 x 2 + 36 x, (1, 4)

(b)

f ( x) = 6 x 3 3 x 3 , (1, 2)

(c)

( x 2 ) 3 , (2, )
f ( x) =

(d)

f ( x) = sin x + cos x, (, )

(e)

2
2
f ( x) = ( sin x ) 3 , ,
6 6

ACTIVITY 10.3
Please visit the website to see some examples and the activities given
http://www.fsas.upm.edu.my/~suriah/bab4.html

162 X TOPIC 10 EXTREMUM PROBLEM AND THE TRACING OF A GRAPH

Now consider the following theorem and its proof:

The Theorem of Local Extremum


If f (c) is a local maximum or a local minimum, then (c, f (c)) is a critical
point.

Proof
1.

Suppose that f is not differentiable at x = c. Then (c, f (c)) is a critical point


and the theorem is satisfied.

2.

Suppose that f is differentiable at x = c. Then f (c) = lim


x c

f ( x ) f (c )
xc

exists. If f (c) = 0, then by definition (c, f (c)) is a critical point.


If f (c) > 0, then lim
x c

f ( x ) f (c )
> 0.
xc

This implies that there is an open interval I containing c such that

f ( x ) f (c )
>0
xc

for every x c in I.
Consequently, this results in two situations of the following:
(i)

f ( x) f (c) > 0 and ( x c) > 0

(ii)

f ( x) f (c) > 0 and ( x c) < 0

for every x c in I .

The situation (i) states that f (x) > f (c) with x to the right of c, and this shows that
f (c) is not the local maximum of f.
The situation (ii) states that f (x) < f (c) with x to the left of c, and this shows that
f (c) is not the local minimum of f. Therefore, the assumption f (c) > 0 is a
contradiction with the statement f (c) is a relative extremum.
Similarly, the assumption f (c) < 0 results in a similar contradiction.
It should be remembered that not at every critical point there occurs a local
1
3

maximum or a local minimum. For example, for f ( x) = x , (0,0) is a critical

TOPIC 10

EXTREMUM PROBLEM AND THE TRACING OF A GRAPH W 163

point since f (0) does not exist but f (0) is not a local extremum because at every
open interval I that contains 0,
f (0) > f (x) if x is to the left of 0 such that f (0) is not a local minimum.
f (0) < f (x) if x is to the right of 0 such that f (0) is not a local maximum.

Remark: In other words, the Local Extremum Theorem exerts that the critical
point is only a candidate of local extremum point but local extremum cannot
happens at other point than the critical point.
In the case of a differentiable function, the Extremum Value Theorem gives us the
following:
If f (c) is a local extremum of f, then f (c) = 0.

Steps of Determining Extremum


As a summary, in order to determine the extremum (absolute) of a function f in a
closed interval [a,b] , the following steps are suggested:
(a)

Obtain all the critical points (c, f (c)) of f in the open interval (a, b).

(b)

Evaluate the value of f at every end point of the interval [a, b] , that is f (a)
and f (b).

(c)

The smallest value of the set { f (c), f (a), f (b)} is the minimum and the
largest is the maximum.

We shall now study the following examples of determining the extremum value.

Example 10.4
2

Determine the extremum of f ( x) = 2 x 3 x 3 in the closed interval [1,2].

Solution:
The derivative of f is f' ( x) = 2 2 x

1
3

=2

1
3

x 1
x

1
3

, x 0.

164 X TOPIC 10 EXTREMUM PROBLEM AND THE TRACING OF A GRAPH

Note that f (1) = 0 and f (0) does not exist. Therefore, there are two critical points
in the interval (1,2) they are (0, f (0)) and (0, f (1)), namely (0,0) and
(1,1). The values of f at the end points of the interval [1,2] are

f (1) = 5 and f (2) = 4 3 3 4 0.76.


By comparing the values of f (1), f (2), f (1) and f (0), we obtain f (1) = 5 is
the minimum and f (0) = 0 is the maximum.

Example 10.5
The operation cost of a petroleum tanker when it is driven at the speed of km/hr.
u

is estimated at 30 + cents per kilometer. By assuming the speed limit is


2

45 u 80, determine the speed that minimized the cost of delivering the
petroleum if the driver is paid RM24.5 per hour.
Solution:
Let the distance traveled be s km. Total cost (cents)
K (u ) = cost of driver + operation cost
=

s
u

(2450) + s 30 +
2
u

s
= 2450 s u 1 + 30 s + u
2
Therefore,
s
2
s
= 2450 s u 1 + 30 s + u
2

K' (u ) = 2450 s u 2 + 0 +

and

K' (u ) = 0 if

or

u = 70

2450 s s
=
u2
2

TOPIC 10

at

EXTREMUM PROBLEM AND THE TRACING OF A GRAPH W 165

u = 70, K (70) =

2450
s
s + 30 s + 70 = 100 s
70
2

u = 45, K (45) =

2450
s
s + 30 s + 45 106.944 s
45
2

u = 80, K (80) =

2450
s
s + 30 s + 80 = 100.625s
80
2

Hence, the speed that minimised the cost of delivery is 70 km per hour.

EXERCISE 10.2
Determine the extremum of each of the function in Exercise 10.1 but
with the following interval.
(a)

[1, 4] ,

(b)

[ 1, 2] ,

(c)

[ 2, ] ,

(d)

[ , ] ,

(e)

2
6 , 3 .

166 X TOPIC 10 EXTREMUM PROBLEM AND THE TRACING OF A GRAPH

10.2

GRAPH TRACING

Consider Figure 10.3 which shows that f is first increasing but then decreasing as
x is greater than a.

Figure 10.3

Notice initially, that as x moves to the right the value of f (x) increases. We obtain
the following definition:

Definition
Suppose f is defined in the interval I. The function f is said to be
(i)

increasing in I if for each pair of x1 and x2 in I,

x2
(ii)

x1 implies that f ( x2 ) > f ( x1 );

decreasing in I if for each pair of x1 and x2 in I,

x2

x1 implies that f ( x2 ) < f ( x1 ).

What is the relation between the increasing or decreasing property of a function f


with its derivative?
Since we are interested in the derivative of f, therefore we have to assume
that f is differentiable. Now, we state and prove the following theorem.

TOPIC 10

EXTREMUM PROBLEM AND THE TRACING OF A GRAPH W 167

Theorem
Suppose f is continuous in the closed interval [a,b] and differentiable in the
open interval (a,b).
1.

If f ( x) > 0x ( a, b ) then f is increasing in the closed interval [a,b].

2.

If f ( x) < 0x ( a, b ) then f is decreasing in the closed interval [a,b].

3.

If f ( x) = 0x ( a, b ) then f is a constant in the closed interval [a,b].

Proof 1
Suppose f ( x) > 0x ( a, b ) and x1, x2 be any two points in the open interval
(a,b) where x2 > x1.
To prove f is increasing in the closed interval [a,b] we only have to show that
f ( x2 ) > f ( x1 ) .
Since f satisfies the hypothesis of the Mean Value Theorem in the closed interval
[ x1 , x2 ] then there exists c ( x1 , x2 ) such that
f (c) =

f ( x2 ) f ( x1 )
x2 x1

However, f (c) 0 and x2 x1 > 0 , therefore it is proved that f ( x2 ) f ( x1 ) > 0 ,


that is, f is increasing in the closed interval [ x1 , x2 ] where x1 , x2 are any points in
the closed interval [a,b]. This completes the proof.

ACTIVITY 10.4
Prove the Second Theorem.

168 X TOPIC 10 EXTREMUM PROBLEM AND THE TRACING OF A GRAPH

Proof 3
Suppose that f ( x) = 0x ( a, b ) but f is not constant in the closed interval
[a,b], that is there exist two distinct points x1 , x2 in the open interval (a,b) such

that f ( x1 ) f ( x2 ) . This means that f ( x2 ) f ( x1 ) 0. Since f satisfies the


hypothesis of the Mean Value Theorem, then there exists c ( a, b ) such that
f (c) =

f ( x2 ) f ( x1 )
0.
x2 x1

This is in contradiction with the supposition that f ( x) = 0x ( a, b ) . Thus we


conclude that f is a constant in the closed interval [a,b].
In section 10.1 we have learnt that a critical point is a candidate of the local
extremum point and it does not necessarily happen that there is local maximum or
local minimum at the critical point.
The following theorem gives further detail of the above using the first order
derivative.

The First Derivative Test Theorem


Suppose that f. is continuous in an open interval (a,b) and (c, f (c)) its critical
point.
1.

If f ( x) > 0x ( a, c ) and

f ( x) < 0x ( c, b ) , then f (c) is a local

maximum of f.
2.

If f ( x) < 0x ( a, c ) and f ( x) > 0x ( c, b ) , then f (c) is a local


minimum of f.

3.

If f (x) has the same sign both to the left and to the right of c, then f (c) is
not a local extremum of f.

We shall only give the proof of 2.

TOPIC 10

EXTREMUM PROBLEM AND THE TRACING OF A GRAPH W 169

Since, f ( x) < 0x ( a, c ) therefore, f decreases in the closed interval [a,c]. In


particular, f decreases in the interval (a,c]. Similarly, since f' ( x) > 0x ( c, b ) ,
then f increases in the interval [c,b). Consequently, f (c) < f (x) for every x in the
open interval (a,b), except at x = c. Therefore, f (c) is a local minimum of f.
We shall now study the application of the First Derivative Test in the
determination of the local extremum.

Example 10.6
f ( x) = x 2sin x in ( 0, 2 ) .
Solution:
Clearly, f is continuous in the open interval
differentiable with f ( x) = x 2 cos x. Notice that

( 0, 2 ) .

Furthermore, f is

1
5
f ( x) = 0 if cos x = , that is x = , .
2
3 3

Therefore, the critical points of f are ,


3

5

f and ,
3
3

5
f .
3


In the open interval 0, we have f (x) < 0, that is f is decreasing.
3
5
In the open interval , we have f (x) > 0, that is f is increasing.
3 3
5

In the open interval , 2 we have f (x) < 0, that is f is decreasing.


3

Based on the First Derivative Test,


f = 3 is a local minimum while
3 3

5 5
f =
3 is a local maximum.
3 3

170 X TOPIC 10 EXTREMUM PROBLEM AND THE TRACING OF A GRAPH

Thus far we have learnt that if f is positive in a certain interval then f is


increasing. All the three functions f (x) = x, g(x) = x3 and h( x) = x are
increasing in the closed interval [0,1]. However, in the closed interval [0,1] the
functions f, g and h have different concavity.

Figure 10.4

Definition
Suppose f is differentiable in an open interval I. The graph of f is said to
concave upward in the interval I if f is increasing and concave downward if f
is decreasing in I.

See Figure 10.4.


Figure 10.5a illustrates the slope of the tangent line of f at x, that is f (x) is
increasing. As we move towards the right, the value of f (x) becomes large. While
in Figure 10.5b, as we move to the right, the value of f (x) becomes small.

TOPIC 10

EXTREMUM PROBLEM AND THE TRACING OF A GRAPH W 171

Figure 10.5

Example 10.7

1 3
x x concaves upward in the open interval (0,). This is
3
because f (x) = x2 1 increases in the open interval (0,), that is as the value of x
becomes large, the value of f (x) becomes large.
The graph of f ( x) =

In the open interval (,0), as the value of x becomes large, the value of f (x)
becomes small. Therefore, f (x) decreases in the open interval (,0).
The concavity of a function can be determined using the second derivative as
given in the following theorem.

Theorem
Suppose that the second derivative of f exists in an open interval I.
1.

If f ( x) 0 x I , then the graph of f is concave upward in I.

2.

If f ( x) 0 x I , then the graph of f is concave downward in I.

172 X TOPIC 10 EXTREMUM PROBLEM AND THE TRACING OF A GRAPH

Example 10.8

1 3
x x concaves upward in the open interval (0,) since
3
f (x) = 2x is positive for every x in the open interval (0,).
The graph of f ( x) =

In the open interval (,0), f (x) < 0. Therefore, the graph of f is concave
downward in the open interval (,0).
Whenever we involve the second derivative we need to distinguish the point of
inflection as we have distinguished the critical point in our earlier study of the
first derivative.
Remember that the critical point is the point at which the value of f (x) is zero or
does not exist. However, the point of inflection is the point at which the concavity
of the graph of f changes. It does not mean that neither the value of f (x) is zero
nor that f (x) does not exist.
See Figure 10.6 which illustrates three types of point of inflection. Note also that
the tangent line at a point of inflection if it exists cuts the graph at the point of
inflection.

Figure 10.6

One important point that is expected from Figure 10.6 is the following theorem:

Theorem
If (c, f (c)) is an inflection point of f, then f (c) = 0 or f (c) does not exist.

TOPIC 10

EXTREMUM PROBLEM AND THE TRACING OF A GRAPH W 173

Remember that the point at which f (c) is zero or does not exist is only a
candidate of the inflection point and the point of inflection does not occur except
at the point where f (c) is zero or does not exist.
Consider the graph of f (x) = x4 as illustrated in Figure 10.7 which clearly shows
that (0,0) is not an inflection point even though f (x) = 12x2 is zero at that point.

Figure 10.7

We have just learnt to use the second derivative to determine the concavity of the
graph of a function. The second derivative can also be used to test the local
extremum as an alternative of the First Derivative Test.

Theorem (Second Derivative Test)


Suppose that f (c) = 0 and f (x) exists for every x in the open interval
containing c.
1.

If f (c) > 0 then f (c) is a local minimum.

2.

If f (c) < 0 then f (c) is a local maximum.

Note that the Second Derivative Test cannot be used if f (c) = 0. Therefore, in
this case we use the First Derivative Test.
Now we prove the second Theorem (Second Derivative Test) above.

174 X TOPIC 10 EXTREMUM PROBLEM AND THE TRACING OF A GRAPH

Proof of 2
If f (c) = 0 and f (x) < 0, there exists an interval I containing c such that

f ( x) f (c) f ( x)
=
< 0 for every x c in I .
xc
xc
This is because

f (c) = lim
x c

f ( x) f (c)
< 0.
xc

If x < c then f (x) > 0 and if x > c then f (c) < 0. According to the First Derivative
Test Theorem, f (c) is a local maximum.

Example 10.9

1 3
x x in the open interval (, ) and mark the
3
critical point, the point of inflection and the local extremum, if they exist.

Sketch the graph of f ( x) =

Solution:
x
x
f ( x) = ( x 2 3) = x 3 x + 3
3
3
2
f ( x) = x 1 = ( x 1)( x + 1)

At x = 0,

)(

3 and 3 the graph of f passes the x-axis.

2
2

There are two critical points, 1, and 1, . In the open intervals (,1)
3
3

and (1,), f is increasing since f (x) > 0. While in the open interval (1,1), f
2

decreasing since f (x) < 0. At the point 1, , f (x) = 2x is positive. Therefore


3

2
according to the Second Derivative Test, f (1) = is a local minimum. Next,
3
2
since f (1) = 2 < 0, then f (1) = is a local maximum.
3

TOPIC 10

EXTREMUM PROBLEM AND THE TRACING OF A GRAPH W 175

Furthermore, f (x) > 0 in the open interval (0,), therefore the graph of f is
concave upward in the interval (0,). In the open interval (,0), the graph of f
is concave downward since f (x) < 0. Next, the point (0,0) is a point of inflection
as f (0) = 0. Thus, we have the graph of f as given in Figure 10.8.

Figure 10.8

In the tracing of the graph of y = f (x) as in example 10.9, we obtain the following
information:
1.

Domain of f.

2.

The intercepts on the x and y axis.

3.

The critical point.

4.

The intervals where the function is increasing/decreasing.

5.

The local extremum points.

6.

The intervals where is concave upward/downward.

7.

The inflection point.

In the following example, we show the tracing of the graph of a given function
with particular attention on the information highlighted above.

176 X TOPIC 10 EXTREMUM PROBLEM AND THE TRACING OF A GRAPH

Example 10.10
1

Sketch the graph of f ( x) = 4 x 3 x 3 .

Solution:
1.
The domain of f is the set of all real numbers.
2.

3.

The intercepts on the x-axis are (0,0) and (4,0).


The intercept on the y-axis is (0,0).
f' ( x) =

4 23 4 13 4 (1 x)
x x
, x 0. The critical points are (0,0) and
2
3
3
3
3
x

(1,3).
4.

In the open intervals (,0) and (0,1), f (x) > 0. Therefore, f is increasing in
the interval (,1]. In the open intervals (1,), f (x) < 0. Hence, f is
decreasing in the interval [1,).

5.

f (1) = 3 is a local maximum based on information 4 above. (It can also be


based on the fact that f (1) < 0 and f (1) = 0.

6.

7.

8 53 4 23
4 (2 + x)
f ( x) = x x =
5
9
9
9
x3
In the open interval (,2), f (x) < 0; the graph of f is concave downward.
In the open interval (2,0), f (x) > 0; the graph of f is concave upward.
In the open interval (0,), f (x) < 0; the graph of f is concave downward.
1
4

The inflection points are (0, 0) and 2, 4 ( 2 ) 3 ( 2 ) 3 . The graph of f is

given in Figure 10.9.

TOPIC 10

EXTREMUM PROBLEM AND THE TRACING OF A GRAPH W 177

Figure 10.9

EXERCISE 10.3
Do the similar analysis as in example 10.10 above for the following
functions and sketch their respective graphs.
(a)

f ( x) = 2 x 3 3 x 10,
x ( x 5)
,
4
2

(b)

f ( x) =

(c)

f ( x) = x 2 ( x 2 1) ,

(d)

f ( x) = 2 cos x + sin 2 x,

(e)

f ( x) = 3x 4 + 4 x3 ,

(f)

f ( x) = e x ,

(g)

f ( x) = 3 x 4 + 4 x 3 12 x 2 + 2.

178 X TOPIC 10 EXTREMUM PROBLEM AND THE TRACING OF A GRAPH

10.3

APPLIED EXTREMUM PROBLEM

Sometimes the problems to be solved are not stated in clear forms. For example
the extremum of a function may not be implicit. In such cases, the specific
formulas have to be first derived.

Example 10.11
Determine the size of a rectangular shaped lot of land whose maximum land area
can be fenced by a fence of length 500 meter.
Solution:
Suppose x as the length (in meter) and y as the width (in meter) of the rectangular
lot. Let also L as the land area to be fenced. Therefore, L = xy m2. But
2x + 2y = 500 or y = 250 x.
Therefore,
L(x) = x(250 x),

0 x 250.

Our problem now is to determine the maximum value of L in the closed interval
[0,250]. Since L (a polynomial) is continuous in the closed interval [0, 250], then
according to the Extremum Value Theorem, L attains its maximum in the
specified interval.
The maximum value of L may be at the end points of the interval or at the critical
point.
Note that L(x) = 250 2x and the value is zero at x = 125. At this point, we
have L(125) = 2 < 0. Therefore, according to the Second Derivative Theorem,
L(125) is a local maximum.
Since at the end points of the interval, L(0) = L(125) = 0 then L(125) is an
absolute maximum. Hence, the rectangular shaped land has a maximum area if it
is a square of length 125 m.

TOPIC 10

EXTREMUM PROBLEM AND THE TRACING OF A GRAPH W 179

Figure 10.10

Example 10.12
An oil platform N, is situated 10 km away from the shore as shown in
Figure 10.10. If the cost of the installation of pipe in the mainland is RM0.5
million per km and RM1million per km in the sea, suggest the pipe installation
that minimised the cost of channeling oil to the point Q.
Solution:
Let x = distance from O to P
K = cost (millions of RM) of pipe installation.
Then
K ( x) = (16 x)(0.5) +
K ( x) = 0.5 +

(x

x
x + 100
2

+ 100 ) (1),

0 x 16

By setting K(x) = 0, we obtain 4x2 = x2 + 100. Hence, we have x =


critical point is only at x =

Next, K" ( x) =
minimum.

10
. The
3

10
10
and K
= 16.66.
3
3

100
10
10
> 0. Then, K
and K"

= 16.66 is a local
x + 100
3
3
2

180 X TOPIC 10 EXTREMUM PROBLEM AND THE TRACING OF A GRAPH

10
Since K(0) = 18 and K(16) = 18.87 then K
is minimum (absolute).
3
10
5.7735 km.
Thus it is suggested that x =
3

Example 10.12
A firm plans to manufacture a cylindrical shaped drinking can of volume 320 ml
The base and the cover are made of a thicker layer cost 10 cents per cm square
while the side from a thinner layer costs 6 cents per cm square. Determine the size
of the drinking can so that the manufacturing cost is minimised.
Solution
Let j and t be the respective radius and height of the cylinder (in cm). The cost of
the base and the cover are 2 j2(10) cents and the cost of the side is2 jt(6) cents.
Thus the total cost is
K = 20 j2 + 12 jt.
But the volume is

j 2t = 320, that is t =

By substituting t =

320
.
j2

320
in K, we obtain
j2

320
3840
K ( j ) = 20 j 2 + 12 j 2 2 = 20 j 2 +
,
j
j
Next
K' ( j ) = 40 j

3840 40 j 3 3840
.
=
j2
j2

j > 0.

TOPIC 10

EXTREMUM PROBLEM AND THE TRACING OF A GRAPH W 181

At the critical point, K( j) = 0, that is


40j 3 = 3840 or j =

96
3.126 cm

As a check that j = 3.126 cm minimised the cost, we need to show that


K(3.126) > 0. In fact,
K" ( j ) = 40 +

7680
> 0 for any j 0.
j3

This also shows that the graph of K is concave upward.


Thus the manufacturing cost is minimum if the radius 3.126 cm and the
height

t=

320
10.428 cm
j3

EXERCISE 10.4
1.

A farmer plans to fence his rectangular shaped land which is


neighboring a straight canal. He does not need to fence along
the canal. He has a fence of length 1000 meter. What is the size
of the rectangle that may be fenced to cover the maximum area?

2.

Determine the point on the line 6x + y = 9 which is nearest to


the point (3,1).

We have learnt about local extremum and the determination of a critical point.

We have also being exposed with the sketching of a graph and the method of
solving applied extremum problems.

T o p ic

LHospitals
Rule

11

LEARNING OUTCOMES
By the end of this topic, you should be able to:
Apply the LHospitals Rule to solve the problem involving limits.

INTRODUCTION

We have learnt about limit and continuity. In order to obtain the limit of a certain
function, especially the exponential function, we can use the process of
differentiation if the usual method cannot be used. In such case, we use the
method called the LHospitals Rule.

11.1

DETERMINATION OF LIMIT

Now we shall learn the application of derivative in mathematics itself, especially


in the determination of limit.
Earlier, in Unit 1, we have learnt for example

(
(

)
)

x2 x
x 2 x lim
22 2 2
x2
=
=
= ,
lim 2
x2 x 1
22 1 3
lim x 2 1
x2

but
lim
x 1

x2 x
x2 1

TOPIC 11

LHOSPITALS RULE W 183

cannot be determined using the method illustrated above since the limit of the
denominator is zero. However, we are able to obtain the limit using the
cancellation technique as follows:

lim
x 1

x ( x 1)
x2 x
x
1
= lim
= lim
= .
2
x 1 x 1 ( x 1)( x + 1) x 1 ( x + 1) 2

We have also seen the geometrical method, for example,

sin x
= 1.
x 0
x

lim

Now, how about the determination of the following limit


lim+

x 1

ln x
ln x
and lim
?
x ( x 1)
( x 1)

Most likely we are not able to obtain the limits using the methods which we have
learnt thus far.

11.2

THE LHOSPITALS RULE METHOD

In general, to obtain the limit lim


xa

1.

lim f ( x) = lim g ( x) = 0;
xa

xa

or
2.

lim f ( x) = lim g ( x) = .
xa

xa

f ( x)
, if both
g ( x)

184 X TOPIC 11 LHOSPITALS RULE

we have

LHospitals Rule
Suppose f and g are differentiable and g ( x) 0 about a (except possibly at a).
Suppose also

lim f ( x) = 0 and lim g ( x) = 0


xa

xa

or

lim f ( x) = and lim g ( x) = .


xa

Then lim
xa

xa

f ( x)
f ( x)
= lim
g ( x) x a g ( x) ,

provided the final limit exists (even if it is or ).

The following examples illustrate the use of the LHospitals Rule in the
determination of the limit of a function.

Example 11.1

lim x 2 x = 0 and lim x 2 1 0


x 1

x 1

Therefore,

(x
lim
(x

x 1

d
) = lim dx ( x
d
1)
(x

x 1

dx

Next,

( 2 x 1) 1
( 2 x 1) = lim
x 1
= .
x 1 ( 2 x )
lim ( 2 x )
2
x 1

lim

( 2 x 1) .
x 1 ( 2 x )

= lim

TOPIC 11

LHOSPITALS RULE W 185

Example 11.2

lim sin x = 0 and lim x = 0.


x 0

x 0

So

sin x
cos x
= lim
= 1.
x 0
x 0
1
x

lim
Example 11.3

lim ln x = 0 and lim+ ( x 1) = 0.

x 1+

x 1

So
1
ln x
= lim+ x = 1.
lim
x 1+ x 1
x 1 1
In actual fact, the notation x a in the LHospitals Rule above can be
replaced with x a+, x a, x or x .
Example 11.4

lim ln x = and lim( x 1) = .


x

So
1
ln x
= lim x = 0.
lim
x x 1
x 1
ACTIVITY 11.1

ds
is the rate of
dt
change of s in inches per minute, how are we going to determine the
rate of change of the area?
If it is given the length of a square is s feet and

186 X TOPIC 11 LHOSPITALS RULE

EXERCISE 11.1
1.

Check whether the LHospitals Rule can be used or otherwise in the


following problems and solve the problems where applicable.
(a)

(c)
(e)
2.

1 sin x
,
cos x
x

sin 2 x
,
x 0
x

(b)

x3
,
x 0 e x 1

(d)

lim

x+3
,
x+5

(f)

lim

x
.
ex

lim

lim

lim

x 0+

ln x
,
cos ecx

lim
2

x 0

Evaluate

(a)

lim xe x ,

(b)

lim x sin

(c)

lim+ tan x ln x,

(d)

1
1
lim x ,
x 0 x
e 1

(e)

x 0

xe x
.
x 0 1 e x

lim

We have learnt about the LHospitals Rule in solving the problem involving
limits.

This rule is used whenever


lim
xa

f ( x)
f ( x) = lim g ( x) = 0;
where lim
xa
xa
g ( x)

or

lim f ( x) = lim g ( x) = .
xa

xa

Answers
TOPIC 1: FUNCTION AND GRAPH
Exercise 1.1
(a)

function

(b)

equation

(c)

equation

(d)

equation

Exercise 1.2
x2
f ( x) = x x = 2
x

, x0
, x<0

Exercise 1.3
1.

(a)

symmetrical about the y-axis

(b)

symmetrical about the origin

(c)

symmetrical about the origin

(d)

not symmetrical.

188 X

ANSWERS

2.

TOPIC 2: LIMIT
Exercise 2.1
Any number in the interval (0,6) except at a = 4.

Exercise 2.2
(a)

(b)

(c)

(d)

1
2

Exercise 2.3
1.

4.

2.

(a)

1.66

(b)

(c)

the value of x that is closer to zero should give a better answer.


However, in this case because of the truncation problem, for small
values of x, the values of x and sin x are assumed the same. This gives
1
the answer as zero. The actual answer is .
6

ANSWERS

Exercise 2.4
(a)

( 3x 7 ) 5 < 0.3 3 ( x 4 ) < 0.3


x 4 < 0.1 0.1 < x 4 < 0.1
3.9 < x < 4.1

(b)

( 3x 7 ) 5 < 0.09 3 ( x 4 ) < 0.309


x 4 < 0.03 0.03 < x 4 < 0.03
3.97 < x < 4.03

(c)

( 3x 7 ) 5 < 0.00012 3 ( x 4 ) < 0.00012


3.99996 < x < 4.00004

Exercise 2.5
(a)

Given an arbitrary > 0 choose = 4 such that


x
1
2 = x 8 < provided 0 < x 8 <
4
4

(b)

Given an arbitrary > 0 choose =

(c)

Given an arbitrary > 0 choose =

such that
4
4 x 12 = 4 x 3 < provided 0 < x 3 <

such that
4
3 ( x 7 ) 5 = 3 x 4 < provided 0 < x 4 <

Exercise 2.6
(a)

116

(b)

3
4

(c)

1
2 3

(d)

1
8

W 189

190 X

ANSWERS

Exercise 2.7
1.

(a)

2
3

(b)

2
5

2.

(a)

(b)

(c)

2
5

Exercise 2.8
1.

(a)

(b)

(c)

(d)

(e)

(f)

2.

(a)

32

(b)

100

3.

(a)

1
1
1
0 = < x >
x
x

Given a positive numbers > 0 choose a number N =

such that

1
0 < provided x > N .
x
(b)

1
1
1
1 =
< x +1 >

x +1
x +1
Given a positive numbers > 0 choose a number M =

1
0 < provided x > M .
x
(c)

Given M > 0 choose =

1
such that
M

1
> M provided 0 > x 0 < .
x

such that

ANSWERS

W 191

(d)

1 4
Given N < 0 choose = such that
N
1
4 < N provided 0 > x 0 < .
x

TOPIC 3: CONTINUITY
Exercise 3.1
1.

2.

(a)

no

(b)

x = l and x = l

(c)

x = n, n integer

(d)

x=2

(a)

lim f ( x) = 4k and lim+ f ( x) = 4 + k

x 2

x2

4
For the function to be continuous at x = 2; 4k = 4 + k that is k = .
3
(b)

1
lim x 2 cos = 0 according to the Sandwich Theorem since
x 0
x
1
x 2 x 2 cos x 2 and further lim x 2 = 0 . In order for the
x 0
x
function to be continuous at x = 0, lim f ( x) = f (0) that is k = 0.
x 0

Exercise 3.2
1.

Suppose f (x) = 4x3 6x2 + 3x 2; f is continuous.


f (l) = l and f (2) = l2. Furthermore, 0 is between l and l2.
According to the Mid Value Theorem, there is a c (1,2) such that
f (c) = 4c3 6c2 + 3c 2 = 0. This shows that c is a root of the equation
f (c) = 4x3 6x2 + 3x 2 = 0.

2.

The function f is continuous, say in the interval [0,3], f (0) = 0 and


f (3) =2l.
Furthermore, 0 < l0 < 2l. According to the Mid Value Theorem, there is a
c (0, 3) such that f (c) = c3 c2 + c = l0.

192 X

ANSWERS

TOPIC 4: TANGENT LINE AND DERIVATIVE


Exercise 4.1
(a)

s (1) s (0)
= 5;
1 0

(b)

v = lim
t 0

s (2) s (1)
= 5;
2 1

s (2) s (0)
=0.
20

s (t ) s (0)
10t 5t 2 0
= lim
= lim (10 5t ) = 10
t 0
t 0
t 0
t 0

10t 5t 2 ) (10 5 )
(
s (t ) s (1)
v = lim
= lim
t 1
t 1
t 1
t 1

(10 ( t 1) 5 (t 1)) = lim (10 5 (t + 1) ) = 10 5 ( 2) = 0


= lim
2

t 1

t 1

v = lim
t 2

t 1

10t 5t 2 ( 20 20 )
s (t ) s (2)
= lim
t 2
t2
t2

5t ( 2 t )
10t 5t 2
= lim
= lim ( 5t ) = 10
= lim

t 2
t 2
t 2 t 2 t 2

Exercise 4.2
(a)

10, 0, 10

(b)

y = 10x, y = 5, y + 10 x = 20

ANSWERS

W 193

(c)

Figure

Exercise 4.3
lim
x 1

f ( x) f (1)
(3 + 2 x) (3 + 2)
2( x 1)
= lim
= lim
=2
x 1
x 1
x 1
x 1
x 1

Exercise 4.4
(a)

lim

x 0

f (1 + x) f (1)
3 + 2(1 + x) 5
2x
= lim
= lim
=2
x 0
x 0 x
x
x

f + x
2

lim
x 0
x
lim

x 0

1
1

f
3 + 2 + x 4
2x
2 = lim
2

= lim
=2
x 0
x 0 x
x

f (4 + x) f (4)
3 + 2(4 + x) 11
2x
= lim
= lim
=2

0
x
x
x

(b)

f ( x) = 2

(c)

f ( x) = lim

x 0

f ( x + x) f ( x)
3 + 2( x + x) 3 2 x
2x
= lim
= lim
=2
x 0
x 0 x
x
x

194 X

ANSWERS

Exercise 4.5
At x = a > 0,
g (a ) = lim
x a

xa
g ( x) g (a)
xa
= lim
= lim
=1
x

a
x

a
xa
xa
xa

At x = b < 0,
g (b) = lim
x b

xb
g ( x) g (b)
( x ) ( b )
= lim
= lim
= 1
x b x b
x b
x b
x b

At x = 0,
lim
x 0

x
g ( x) g (0)
= lim
x a x
x0

but
lim

x 0

lim+

x 0

x
x
x
x

= lim

x
= 1
x

= lim+

x
=1
x

x 0

x 0

Exercise 4.6
1.

2.

2
f ( x) = , x 3
x

(a)

f ( x) = x 2 , x = 4

(b)

(c)

f ( x) = 2x 2 , x = c

(d)

f ( x) = x 2 + 2 x, x = 3

(e)

f ( x) = 2x 3 , x = 5

(a)

16e4x

(b)

3 ex

(c)

2e2x

(d)

22 x

ANSWERS

Exercise 4.7
cos( x + x) cos x
cos x(cos x 1)
sin x sin x
= lim
lim
x 0
x 0
x 0
x
x
x
lim

(cos x 1)
sin x
sin x lim
x 0
x 0
x
x

= cos x lim

= cos x(0) sin x(1)


= sin x

TOPIC 5: APPLICATIONS OF INTEGRATION


Exercise 5.1
15x 6

Exercise 5.2
24x + 7

Exercise 5.3
1.

(a)

sec x(1 + x tan x)

(b)

3x 2 4 x 3
(3x 2) 2

(c)

4
(1 + 2 x) 2

(d)

2t 3 t 2 1
t2

(e)

1
1 sin t

(f)

2sec 2 t
(1 tan t ) 2

W 195

196 X

2.

ANSWERS

d u vu uv
=
.
We know that dx v
v2
For u = 1 and v = g we have
d 1 g ( x)(0) 1 g ( x)
g ( x)
=
.

=
2
dx g ( x)
( g ( x ))
( g ( x)) 2
By taking g(x) = xm, m positive integer
d 1
mx m 1
=

= (m) x ( m ) 1.
m
2m
dx x
x
d n
x ) = nx n 1 , n negative integer.
Therefore,
(
dx

Exercise 5.4
(a)

8 ( 3 x 5 + 7 ) (15 x 4 )

(b)

5 ( 2 x 4 3 )

(c)

4 x +
x

(8 x )
3

1 2
x

Exercise 5.5
(a)

2 tan x sec 2 x

(b)

2sin ( 3 x 3 + 6 ) cos ( 3 x3 + 6 )( 9 x 2 )

Exercise 5.6
3 x 2 sin 2 x ( x cos x + sin x )

W 197

ANSWERS

Exercise 5.7
1.

2.

( 3x

(i)

2 x ) 2 ( x 2 + 3) ( 2 x ) ( x 2 + 3) 3 ( 3 x 3 2 x ) ( 9 x 2 2 )
3

( 3 x3 2 x )

12 x 2 ( x 3 + 1)3

1 1
2 + 1 2
x x

(ii)

(iii)

(x

2 x
2

+ 1)

Exercise 5.8
1.

2.

(a)

3 x 2 8 xy
4x2 3

(d)

(a)

9
y 3 = ( x 1)
7

; 7y + 9x = 30

(b)

y 1 = 0 x
2

; y=1

(c)

y 1 =

(d)

y 3 =

(e)

y 0 = 6(x 1)

x
2

a y

(b)

(e)

(c)

y
x

2
( x 4)
17

; 17y + 2x = 25

7
( x 2)
24

; 24y + 7x = 86
; y + 6x = 6

y 2 y sin xy
3 y 2 2 xy + x sin xy

198 X

ANSWERS

TOPIC 6: THEOREM ON DIFFERENTIABLE FUNCTION


Exercise 6.1
1

1
2 = 1.

1.

( f ) 12 = g 12 =

2.

1
1
If ( f 1 ) = x, then f ( x) = .
2
2

1
2

1
1
= , therefore, x = 1.
1+ x 2

Exercise 6.2
0 (2)3
2
, c = 1
1 (1)
3

(a)

f (c) = 3(c 1) 2 =

(b)

1
0
c(c + 2) 2
=
g (c) =
, c = 2 1
(c + 1) 2 1 0

Exercise 6.3
1
3
3

(a)

f (c) = 3c 2 1, c = +

(b)

g (c) = cos c sin c = 0, c =

or c =

5
4

ANSWERS

TOPIC 7: VARIOUS RATES OF CHANGE


Exercise 7.1
O

dx
= 70 km/h
dt
dy
= 90 km/h
dt

z 2 = x2 + y 2 ,

dz 1 dx
dy
= x + y
dt z dt
dt

when x = 0.4 and y = 0.3; dz


1
=
(0.4(70) + 0.3(90)) = 110 km/h
dt 0.5

Exercise 7.2
1.

R=

( R + R2 )( R1R2 + R2 R1 ) R1R2 ( R1 + R2 )
R1 R2
, R = 1
2
R1 + R2
( R1 + R2 )

when R1 = 100, R2 = 80, R1 = 0.2 and R2 = 0.3 .


we obtain R = 0.132 / s
2.

1
L = t k , L = 13, k = 1
2
when
from L =

13 =

1
(t k + t k ) we obtain
2

1
1
(20(1) + t (18)), that is t = cm/s
2
3

W 199

200 X

ANSWERS

TOPIC 8: APPLICATION IN ECONOMICS


Exercise 8.1
1.

C ( x) = 3.25 +

8300 40
+ 2,
x
x3

C (1000) = 11.95
40

C ( x) = 3.25 +

3x
2.

2
3

, C (1000) = 3.383

P ( x) = xp ( x) = 5 x 0.002 x 2 ; P( x) = 5 0.002 x
; C ( x) = 1.1

C ( x) = 3 + 1.1x

K ( x) = P ( x) C ( x) = 3.9 x 0.02 x 2 3; K ( x) = 3.9 0.004 x


x

P( x)

C( x)

K ( x)

800

1.8

1.1

0.7

975

1.1

1.1

1100

0.6

1.1

0.5

TOPIC 9: APPROXIMATION
Exercise 9.1
1.
2.

4
I = r3,
2
(a)

dl = 4 r 2 dr = 4 (8) 2 (0.1) = 80.425 cm3

c 2 = 1202 + 1202 2(120) cos = 28800 28800 cos


= 28800(1 cos ), c = 59.38
2c dc = 28800sin d , dc = 0.58

ANSWERS

(b)

l=

W 201

1
(120)(120) sin = 3451.86
2

1
dl = (120)(120) cos d = 31.59
2

Exercise 9.2
(a)

f (16 + 0.6) f (16) + f (16)(0.6) = 16 +

(b)

f (8 + 0.8) f (8) + f (8)(0.8) = 8 3 +

1
3 8

(c)

f (64 + 1) f (64) + f (64)(1) = 64 3 +

2
3

1
(0.6) = 4.074
2 16
(0.8) = 2.067

2
3 64

(d)

29 = (30 1) = (30 1)

1
3

(1) = 16.167


rad =
rad
180
6 180





f
f + f
= 0.866 + (0.5)
= 0.874
6 180
6
6 180
180
(e)

32 = (30 + 2) = (30 + 2)


rad = + rad
180
6 90





f + f + f = 0.500 + 0.866 = 0.530
6 90
6
6 90
90

202 X

ANSWERS

Exercise 9.3
1.

(a)

1
3

1 23
2 53
10 83
f ( x) = x , f ( x) = x , f ( x) = x , f ( x) =
x
3
9
27
f iv ( x) =

80 113 v
80 11 143
x , f ( x) =
x
81
81 3

1
2
10 iv
80
880
f (1) = 1, f (1) = , f (1) = , f (1) =
, f (1) = , f v (1) =
3
9
27
81
243
1 80
1
1 2
1 10
1 + ( x 1) + ( x 1) 2 + ( x 1)3 + ( x 1) 4 +
24 81
3
2 9
6 27
1 880
5

( x 1)
20 243

(b)

2 ( 3 x 2 1)
1
2x
, f ( x) =
, f ( x) =
f ( x) =
3
2 2
1 + x2
+
1
x
( )
(1 + x2 )
f (0) = 1, f (0) = 0, f (0) = 2
1+ 0 +

1
( 2 ) x 2 = 1 x 2
2

1 2 1 4 1 6
x + x + x
2
4!
6!

(c)

1+

(d)

1
1
1
1 x2 + x4 x6
2
4!
6!

(a)

(1 + 0.3) 3 1 +

2.

1
1 10
1
(0.3) + (0.3) 2 + (0.3)3
3
6 27
9

1 80
1 880
4
5
(0.3) +

(0.3)
24 81
120 243

= 1.0916

ANSWERS

(b)

1
1 0.22 = 0.96
2
1 + 0.2

(c)

1
1
1
cosh(0.3) 1 + (0.3) 2 + (0.3) 4 + (0.3)6 = 1.0453
2
4!
6!

(d)

1
1
1
cos(0.2) 1 (0.2)2 + (0.2) 4 (0.2)6 = 0.98006
2
4!
6!

Exercise 9.4
1.

2.

(a)

x2 = x1

f ( x1 )
f ( 2)
1
= 2
= 2
= 2.25
f ( x1 )
f ( 2)
4

(b)

x2 = x1

f ( x1 )
f (1.5 )
cos1.5
= 1.5
= 1.5
= 1.579
f ( x1 )
f (1.5 )
sin1.5

(c)

x2 = x1

f ( x1 )
f (7)
3 6 7
=7
=7
= 7.899
3
f ( x1 )
f (7)
1
2 6

x2 = x1

h ( x1 )
1
2x
; h( x) = 3 x 2 ; h ( x1 ) = 1 +
2
h ( x1 )
x +1
(1 + x 2 )

take x1 = 3
1
10 = 2.8936
x2 = 3
6
1 +
100

x3 = x2

h ( x2 )
h ( 2.8939 )
= 2.8936
= 2.8936
h ( x2 )
h ( 2.8936 )

Notice that x3 x2 = 0.0003 < 0.001

W 203

204 X

ANSWERS

TOPIC10: EXTREMUM PROBLEM AND THE TRACING OF


A GRAPH
Exercise 10.1
(a)

f ( x) = 6x2 30x + 36 = 6(x2 5x + 6) = 6(x 2)(x 3)


f (x) = 0 when x = 2 or x = 3
The critical points are (2,28) and (3,27).
1

(b)

f ( x) = 8 x 3

1
x

2
3

1
x

2
3

( 8 x 1) ,

x0

1 9
The critical points are (0,0) and , .
8 8
1

(c)

f ( x) = 8 x 3

1
x

2
3

1
x

2
3

( 8 x 1) ,

23
The critical points are 6,

(d)

x0

, 2, 0
( ).

f (x) = cos x sin x


n

The critical points are ( 8n + 1) 4 , 2 and ( 8n + 5 ) , 2 ; n integer.


4

(e)

f ( x) =

2
3(cos x)

2
3

, cos x 0


The critical points is ,1 .
2

ANSWERS

W 205

Exercise 10.2
(a)
(b)

(c)

f (1) = 23, f (4) = 32. Minimum f (1) = 23, maximum f (4) = 32

9
1
f (1) = 9, f (2) = 11.34. Minimum f =
8
8
Maximum f (2) = 11.34.
f is not continuous at x = 0 and lim f ( x) = ,
x 0

lim f ( x) = . There are no maximum and minimum.

x 0+

(d)

(e)

Maximum f ( 8n + 1) = 2 ; minimum f ( 8n + 5 ) = 2
4
4


2
f = 0.63, f
= 0.91
6
3


maximum f = 1 ; minimum f = 0.63.
2
6

Exercise 10.3
(a)

1.

Domain: all real numbers.

2.

x-intercept: (2,0). y-intercept: (0,10)

3.

f ( x) = 6x2 3 = 3(2x2 1).


1

The critical points:


, 11.4 and
, 8.6 .
2
2

4.

1
1

, and ,
f (x) > 0 in the open intervals
.
2
2

1
1

So f is increasing in the open intervals


, and ,
.
2
2

206 X

ANSWERS

1 1
f ( x) < 0 in the open interval
,
.
2 2

1 1
,
So f is decreasing in the open interval
.
2 2

5.

1
f
= 11.4 is a local minimum.
2
1
f
= 8.6 is a local maximum.
2

6.

f (x) =12x
In the open interval (0,), f (x) > 0; the graph is concave upward.
In the open interval (,0), f (x) < 0; the graph is concave downward.

7.

The inflection point is (0,10)

(a)

1.

Domain: (0,)

2.

x-intercepts: (0,0) and (5,0). y-intercept: (0,0).

ANSWERS

3.

f ( x) =

W 207

5( x 5)( x 1)
, x0
8 x

the critical points are (0,0), (1,4) and (5,0).


4.

f (x) > 0 in the open intervals (5,) and (0,1) . So f is increasing in


the open intervals (0,1) and (5,).
f (x) < 0 in the open interval (1,5) . So f
interval (1,5).

5.

is decreasing in the open

f (1) = 4 is a local maximum.


f (5) = 0 is a minimum.

6.

f ( x) =

5
3x 2 6 x 5)
(
16 x x

In the open interval (0,2.633), f (x) = 0 , so f is concave downward.


In the open interval (2.633,), f (x) = 0, so f is concave upward.
7.

The inflection point is (2.633,2.27)


(b)

208 X

ANSWERS

(c)

(d)

(e)

ANSWERS

W 209

(f)

(g)

Exercise 10.4
1.

2x + y = 1000, y = 1000 2 x
l(x) = x (1000 2x)
l(x) = 1000 4x

Therefore, x = 250, y = 500

210 X

ANSWERS

2.

j 2 ( x) = ( x + 3) 2 + (8 6 x) 2
2 j ( x) j ( x) = 2( x + 3) + 2(8 6 x)(6)
j ( x) =

37 x 45

{( x + 3)

+ (8 6 x) 2 }

45
45
45 63
j = 0 and l > 0. Therefore, ,
37
37
37 37

TOPIC 11: LHOPITALS RULE


Exercise 11.1
1.

sin 2 x
2 cos 2 x
= lim
=2
x 0
x 0
1
x

(a)

lim

(b)

lim
x

1 sin x
cos x
= lim
=0
cos x
x sin x
2

(c)

x3
3x 2
lim x
= lim x = 0
x 0 e 1
x 0 e

(d)

lim

(e)

ln x
1

lim+
= lim+
x 0 cos ec x
x 0
x cot x cos ec

(f)

lim

x 0

x+3 3
=
x+5 5

x
1
= lim x = 0
x
x e
x e

sin x
x

lim
=

=0

x x0+ cot x

ANSWERS

(g)

2.

(a)

(b)

lim

x
1
= lim x =
x
x

e
e

lim xe x = lim
x

lim x sin
x

x
1
= lim x = 0
x
x

e
e

= lim
x

sin

x = lim sin =

(c)

1
2

ln x
x = lim sin x = 0
lim+ tan x ln x = lim+
= lim+

1 x0+
x 0
x 0 cot x
x 0
x
2
sin x

(d)

ex 1 x
1
1
lim x = lim

x 0 x
e 1 x0 x(e x 1)

ex
1 1
= lim x
= lim

=
x 0 e ( x + 2)

x 0 x + 2 2

(e)

xe x
xe x + e x
x +1
lim
= lim
= lim
= 1
x

0
x 0 1 e x
x 0
x
e
1

W 211

MODULE FEEDBACK
MAKLUM BALAS MODUL
Should you have any comment or feedback, you are welcomed to:
1.

E-mail your comment or feedback to modulefeedback@oum.edu.my

OR
2.

Download and fill up the feedback questionnaire from


URL: http://lms.oum.edu.my/ via myVLE
and
e-mail to modulefeedback@oum.edu.my

Thank you.
Centre for Instructional Design and Technology
(Pusat Reka Bentuk Pengajaran dan Teknologi)
Tel No.:

03-27732273

Fax No.:

03-26978702

Anda mungkin juga menyukai